You are on page 1of 42

Contact us : info@onlyias.

com
Visit : dpp.onlyias.in
Contact : +91-7007 931 912

DEDICATED PRELIM PROGRAM(DPP)


UPSC-CSE PRELIMINARY TEST SERIES – 2021/2022
DO NOT OPEN THIS TEST BOOKLET UNTIL YOU ARE ASKED TO DO SO
TEST BOOKLET SERIES

Test Code:

E1
TEST BOOKLET

DPP GENERAL STUDIES


TEST -01

Time Allowed: Two Hours Maximum Marks : 200

INSTRUCTIONS
1. IMMEDIATELY AFTER THE COMMENCEMENT OF THE EXAMINATION, YOU SHOULD CHECK THAT THIS TEST
BOOKLET DOES NOT HAVE ANY UNPRINTED OR TORN OR MISSING PAGES OR ITEMS, ETC.
2. ENCODE CLEARLY THE TEST BOOKLET SERIES CODE A, B, C OR D AS THE CASE MAY BE IN THE
APPROPRIATE PLACE IN THE ANSWER SHEET.
3. You have to enter your Roll Number on the Test Booklet in the Box
provided alongside this line. DO NOT write anything else on the
Test Booklet.
4. This Test Booklet contains 100 questions. Each question is printed in English. Each item comprises four
responses (answers). You will select the response which you want to mark on the Answer Sheet. In case you feel
that there is more than one correct response, mark the response which you consider the best. In any case, choose
ONLY ONE response for each item.
5. All items carry equal marks.
6. Penalty for wrong answers:
THERE WILL BE PENALTY FOR WRONG ANSWERS MARKED BYA CANDIDATE IN THE OBJECTIVE TYPE
QUESTION PAPERS.
I. There are four alternatives for the answer to every question. For each question for which a wrong
answer has been given by the candidate, One-Third of the marks assigned to that question will be
deducted as penalty.
II. If a candidate gives more than one answer, it will be treated as a wrong answer even if one of the given
answers happens to be correct and there will be same penalty as above to that question.
III. If a question is left blank, i.e., no answer is given by the candidate, there will be no penalty for that
question.

DO NOT OPEN THIS TEST BOOKLET UNTIL YOU ARE ASKED TO DO SO


TOPICS COVERD: Historical background, Salient features of constitution, Amendment to constitution,
Preamble, Making of constitution
Contact us : info@onlyias.com

OnlyIAS Nothing Else Visit : dpp.onlyias.in


Contact : +91-7007 931 912

Q.1) Arrange the following functions performed by 2. Liberty cannot be achieved with the absence of
the Constituent Assembly in a chronological order: laws.
1. Adoption of National Flag 3. It provides an opportunity for an individual to
2. Election of Dr. Rajendra Prasad as the President of develop himself/herself fully.
the Assembly 4. Liberty is essential for the successful functioning
3. Adoption of Objective Resolution of democracy.
4. Ratification of India’s membership of the Which of the above statement(s) is/are incorrect?
Commonwealth a) 1 only
Select the correct answer using the code given b) 1 and 2 only
below: c) 2, 3 and 4 only
a) 2-1-4-3 d) None of the above
b) 3-2-1-4
c) 3-1-2-4 Q.5) Which of the following is incorrect about the
d) 2-3-1-4 term ‘Republic’ mentioned in the Preamble?
a) The head of the State is always directly elected.
Q.2) Consider the legislative powers of Constituent b) Vesting of political sovereignty in the people
Assembly: c) All political offices are opened to every citizen
1. The legislative powers were given by the Indian without any discrimination.
Independence Act, 1947. d) None of the above
2. The dual task of Constitution making and
legislation continued till the commencement of Q.6) Which of the following Articles of the
the Constitution. Constitution promote social justice as envisaged in
3. The Constituent Assembly continued as a the Preamble?
provisional Parliament from January 26, 1950. 1. Article 17
Which of the above statement(s) is/are correct? 2. Article 38
a) 1 and 3 only 3. Article 39A
b) 2 only 4. Article 42
c) 2 and 3 only 5. Article 49
d) 1 and 2 only 6. Article 50
Select the correct answer using the code given
Q.3) Consider the following statements about the below:
Objective Resolution moved in the Constituent a) 1, 2 and 4 only
Assembly: b) 2, 3 and 6 only
1. It declared India as a Sovereign, Socialist, Secular c) 1, 2, 3, 5 and 6 only
Republic. d) 1, 2, 3 and 4 only
2. It envisaged to make the people of India as the
source of power and authority of Independent Q.7) Consider the following statements about the
India. Preamble:
3. It envisaged to provide safeguards to minorities, 1. The Preamble has been amended only once so far
backward and tribal areas and other backward under the Prime Ministership of Mrs. Indira
classes. Gandhi.
Which of the statement(s) given above is/are 2. The Supreme Court in the Minerva Mills case has
correct? held that the Preamble is part of the Constitution.
a) 1 and 2 only 3. The original Preamble did not include the word
b) 1 only ‘unity and integrity’ in it.
c) 1, 2 and 3 Which of the statement(s) given is/are correct?
d) 2 and 3 only a) 1 and 3 only
b) 1 only
Q.4) Consider the following statements about the c) 2 and 3 only
term ‘Liberty’ mentioned in the Preamble: d) 3 only
1. The Preamble provides liberty of thought,
expression, opportunity, faith and worship.

TEST 1 1
Contact us : info@onlyias.com

OnlyIAS Nothing Else Visit : dpp.onlyias.in


Contact : +91-7007 931 912

Q.8) Consider the following statements about the 3. Franchise to a limited number of people based on
term ‘Secular’ as mentioned in the Preamble of the property, tax, education is provided in the
Constitution: Government of India Act, 1919.
1. The term was added by the 44th Constitution 4. Voting rights were given to around 10 percent of
Amendment Act. the population as per GOI, 1919.
2. Indian secularism provides for strict separation of 5. Universal Adult Franchise was introduced in the
State and religion. Indian Constitution under Article 326.
3. Article 15, 16 and 44 of the Constitution promotes Select the correct answer from the codes given below:
secularism in India. a) 1, 2 and 5 only
Which of the above statements is/are correct? b) 2, 4 and 5 only
a) 1, 2 and 3 only c) 2, 3 and 5 only
b) 2 and 3 only d) 2, 3 and 4 only
c) 2 only
d) 3 only Q.12) Consider the following statements regarding
the Indian Independence Act, 1947:
Q.9) Select the correct chronological order of the 1. It abolished the office of the General-General of
following events: India and British monarch’s right to veto bills.
1. Lowering of Voting Age from 21 to 18 years. 2. Governor General, in the name of His majesty, had
2. Constitutional recognition to the Panchayati Raj full powers to give assent to the bills.
Institutions. 3. It abolished the office of the Secretary of State.
3. Kihoto Hollohon case Select the correct answer from the codes given below:
4. Setting up of Sarkaria Commission. a) 1 and 3 only
Select the correct answer from the codes given b) 2 and 3 only
below: c) 1 and 2 only
a) 1-4-2-3 d) 1, 2 and 3
b) 4-1-3-2
c) 1-4-3-2 Q.13) Which of the following statements is correct:
d) 4-1-2-3 a) Pitt’s India Act provided for the payment of the
Board of Control charged to the East India
Q.10) Which one of the following is most appropriate company.
regarding the amendment to the constitution. b) Local government was empowered to levy taxes by
a) Constitutional Amendment Acts cannot be the Charter Act of 1813.
introduced in the Rajya Sabha. c) Simon Commission recommended for the
b) Election’s procedure of the Parliament and the continuation of Dyarchy in the provinces.
State Legislatures require assent of the State d) Ramsay MacDonald was called father of the
legislatures. communal electorate for the Communal Award,
c) State legislatures must ratify a Constitutional 1932.
Amendment Bill within 6 months.
d) Changes in Salaries and Allowances of the MPs Q.14) With reference to the Indian Independence Act
amounts to a Constitutional Amendment outside 1947, consider the following statements:
Article 368. 1. The Act designated the Governor-general of India
and provincial governor as constitutional
(nominal) heads of the states.
2. It provided for the governance under the
Q.11) Consider the following statements regarding
provisions of Government of India Act of 1935
elections in India:
strictly without any modifications.
1. Indirect provision for elections introduced in Indian
3. It provided that the Constituent Assembly should
Councils Act, 1861.
also function as a legislative body.
2. Direct elections to majority of members in both the
Which of the statements given above is/are correct?
houses was introduced in the Government of India
a) Only 1
Act, 1919.
b) 1 and 3
c) Only 3
d) 2 and 3.
TEST 1 2
Contact us : info@onlyias.com

OnlyIAS Nothing Else Visit : dpp.onlyias.in


Contact : +91-7007 931 912

2. 2.It means that all citizens of a country have the


Q.15) With reference to the Charter Act of 1833, right to vote, irrespective of caste, creed, religion,
consider the following statements: sex or age.
1. After this Act, the Government of India had 3. It opens up new hopes and vistas for weaker
authority over the entire territorial area sections of the society.
possessed by the British in India. Which of the statements given above are correct?
2. The East India Company became an administrative a) 1 only
body but it also continued as a commercial body. b) 2 only
Which of the statements given above is/are correct? c) 1 and 2 only
a) Only 1 d) 1 and 3 only
b) Only 2
c) Both 1 and 2 Q.19) Consider the following statements with
d) Neither 1 nor 2 respect to the Indian Constitution
1. In India, there is an integrated system of courts
enforcing both central and state laws.
Q.16) Consider the following statements: 2. 2.The scope of judicial review in India is narrower
1. The Constituent Assembly was formed on the than that of what exists in the USA.
basis of Indian Independence Act,1947. 3. The Emergency Provisions in the Constitution
2. The Constituent Assembly was a partly elected converts the federal structure into a unitary one
and partly nominated body. without any formal amendment of the
3. As the number of Princely states were high , Constitution.
majority of the seats were allotted to it. Which of the statements given above are incorrect?
Which of the statements given above is/are correct? a) 1 and 2 only
a) 1 and 2 only b) 2 and 3 only
b) 2 only c) 1 and 3 only
c) 1 and 3 only d) None of the above
d) 1,2 and 3
Q.20) Consider the following pairs about the Indian
Constitution?
Q.17) Which of the following regarding Schedule Provision
Constitutional provisions for Secularism are 1. Fourth
Incorrect? Schedule Allocation
1. 1.Indian concept of Secularism implies a complete of seats in the Lok Sabha to the states.
separation of the religion and the state. 2. Third
2. 2.The term “Secular” was a part of the original schedule Forms of
constitution. oaths and affirmations.
3. The provision of equality of opportunity for all 3. Eight
citizens in matters of public employment reveals schedule Languages
the Secular character of the Indian State. recognised in the Constitution.
Select the answer using the code below- 4. Ninth
a) 1 and 2 only schedule Disqualifica
b) 2 only tion of the members of Parliament
c) 2 and 3 only Select the correct answer using the code below-
d) 3 only a) 1 and 2 only
b) 2 and 3 only
c) 3 and 4 only
d) 1 and 4 only
Q.18) Consider the following statements with
respect to Universal Adult Franchise in India
Q.21) With reference to the Indian constitution,
1. 1.Universal Adult Franchise makes democracy
consider the following statements-
broad- based and upholds the principle of
1. 1.All the provisions of the Indian Constitution can
Equality.
be easily amended by the Parliament.

TEST 1 3
Contact us : info@onlyias.com

OnlyIAS Nothing Else Visit : dpp.onlyias.in


Contact : +91-7007 931 912

2. 2.Some provisions of the Constitution can be a) 1 only


amended by a simple majority of the parliament. b) 2 only
3. The provisions relating to Use of the English c) Both 1 and 2
language in Parliament requires a Special Majority d) Neither 1 nor 2
of Parliament for amendment.
Which of the statements given above is/are correct?
1. 1 and 3 only
2. 2 only Q.26) Consider the following statements:
3. 2 and 3 only 1. The office of the secretary of state was created
4. All of the above before the revolt of 1857.
2. Secretary of State was the member of the British
Q.22) Consider the following statements: Cabinet and headed the Board of Control in India.
1. The Regulating Act of 1773 created the double 3. The office of secretary of state was abolished in the
government system. Government of India Act, 1935.
2. The Pitt’s India Act,1784 made Lord Cornwallis as Select the correct answer using the codes given
the commander-in-chief along with the Governor- below:
General. a) 1 and 2 only
Select the correct answer from the codes given below: b) 2 and 3 only
a) 1 only c) 1, 2 and 3
b) 2 only d) None of the above.
c) Both 1 and 2
d) Neither 1 nor 2 Q.27) Consider the following statements regarding
the creation of Federal Court in India:
Q.23) Consider the following statements regarding 1. It was established as per the provisions of the
the Charter Act of 1793: Government of India Act, 1935
1. The Act extended the trade monopoly of the 2. Sir Elijah Impey was the first Chief Justice of this
Company in India for another period of 20 years. Court.
2. This Act was also termed as the St. Helena Act. Select the correct answer from the codes given below:
3. The strength of the Court of Directors was reduced. a) 1 only
Select the incorrect answer using the codes given b) 2 only
below: c) Both 1 and 2
a) 1 and 2 only d) Neither 1 nor 2
b) 2 and 3 only
c) 1 and 3 only Q.28) Consider the events given below:
d) 1, 2 and 3 1. Introduction of direct election.
2. Abolition of Board of Control
Q.24) Which of the below provisions are correct 3. Separate electorates for the women
regarding the Morley Minto Reforms: 4. Final set up of the Federal Court
a) It introduced bicameralism in the Indian Legislative 5. Vernacular Press Act
Council at the central level. 6. Submission of report by Simon Commission
b) Ordinance making power was given to the Viceroy. Arrange the event above in the correct chronological
c) There was a system of indirect elections for the sequence:
elected members of the councils. a) 1-2-3-4-5-6
d) The Act preceded the formation of the Muslim b) 2-5-4-3-1-6
League. c) 2-5-1-6-3-4
d) 1-5-6-3-1-4
Q.25) Consider the following statements:
1. Charter Act of 1853 made attempts for the Q.29) Consider the following statements:
abolition of slavery in India. 1. The Government of India Act, 1935 provided for
2. Slavery was finally abolished during the tenure of dyarchy at the Province.
Lord Ellenborough. 2. The Government of India Act, 1935 abolished the
Select the correct answer using the codes given Council of India established by Government of
below: India act 1858.

TEST 1 4
Contact us : info@onlyias.com

OnlyIAS Nothing Else Visit : dpp.onlyias.in


Contact : +91-7007 931 912

Select the correct answer from the codes given below: 2. The Constitution assemblies of the dominion
a) 1 only government had no powers to repeal The Indian
b) 2 only Independence Act, 1947.
c) Both 1 and 2 Select the correct answer using the codes given
d) Neither 1 nor 2 below:
a) 1 only
Q.30) Consider the following statements: b) 2 only
1. Division of subjects into three lists- Federal, c) Both 1 and 2
Provincial and Concurrent were provided within d) Neither 1 nor 2
the Government of India Act, 1919.
2. The provision for establishment of a joint public Q.34) Consider the following statements regarding
service commission was given by the Government the Amending Act of 1781:
of India Act, 1935. 1. It was also referred to as the Act of Settlement.
Select the correct answer using the codes given 2. It outlined the jurisdiction of the Supreme court at
below: Calcutta.
a) 1 only 3. It provided that the appeals from the provincial
b) 2 only courts were taken to the Supreme Court.
c) Both 1 and 2 Select the correct answer using the codes given
d) Neither 1 nor 2 below:
a) 1 and 3 only
b) 1 and 2 only
Q.31) Which of the following was the most c) 2 and 3 only
appropriate regarding the development of d) 1, 2 and 3
Constitutional history of India:
a) The National flag of India was adopted by the Q.35) Consider the following statements:
constituent assembly after the independence of 1. Charter Act of 1853 completely abolished the
India in 1947. commercial activities of the Company.
b) Ninth Schedule was added in the Constitution of 2. Charter Act of 1853 introduced local
India to deal with the laws relating to land reforms. representation in the Indian Legislative Council.
c) The Concurrent list of the Indian Constitution has Select the correct answer using the codes given
been borrowed from the Government of India Act, below:
1919. a) 1 only
d) The discussions of the Third Round Table b) 2 only
Conference led to the signing of the Poona Pact. c) Both 1 and 2
d) Neither 1 nor 2
Q.32) Consider the following pairs of principles of
Preamble and Rights that promote those principles: Q.36) Consider the following pairs:
Principle Fundamental Right Act Provision
1.Equality of opportunity Article 16 1. Charter Act, 1793: Started Dual
system of Government
2.Liberty of belief Article 25 2. Charter Act of 1853: Introduce open
3.Dignity of individual Article 17 competition for selection of civil servant
4.Social Justice Article 21A 3. Government of India Act, 1935: Provincial
Which of the above pairs are correctly matched? Autonomy
a) 2 and 3 only Which of the above pairs is/are correctly matched?
b) 1 and 2 only a) 1 and 2 only
c) 1, 2 and 3 only b) 2 and 3 only
d) 1, 2, 3 and 4 c) 1, 2 and 3 only
d) None of the above
Q.33) Consider the following statements:
1. The Constituent Assembly was formed as per the Q.37) Consider the following statements:
provisions of the Mountbatten Plan. 1. The portfolio system was recognised by the Indian
Council Act, 1861.
TEST 1 5
Contact us : info@onlyias.com

OnlyIAS Nothing Else Visit : dpp.onlyias.in


Contact : +91-7007 931 912

2. Lee Commission of 1923 dealt with the codification Select the correct answer using the codes given
of the Indian laws. below:
Select the correct answer using the codes given a) 1 only
below: b) 2 only
a) 1 only c) Both 1 and 2
b) 2 only d) Neither 1 nor 2
c) 1 and 2
d) Neither 1 nor 2
Q.42) Consider the following statements:
1. A constitutional amendment bill can be introduced
only in the Lok Sabha.
Q.38) Consider the following statements: 2. States do not have any power to initiate a
1. 52nd Constitutional Amendment Act added Tenth constitutional amendment.
Schedule to the Constitution. 3. The President can either accept or withhold his
2. 86th Constitutional Amendment Act provided assent to the Constitutional amendment bill but
special provisions for the backward classes in all cannot return it.
educational institutions. 4. A constitutional amendment bill seeking to amend
Select the correct answer from the codes given below:
the federal provisions needs ratification by 50
a) 1 only percent of the states.
b) 2 only Select the correct answer from the codes given below:
c) Both 1 and 2 a) 1 and 4 only
d) Neither 1 nor 2 b) 2 and 4 only
c) 2 and 3 only
Q.39) On a particular day in Lok Sabha, there are 541 d) 1 and 3 only
members present of which 8 abstain from voting. A
bill seeking to make changes in Article 50 has been
taken up. Select the number of votes required in favor Q.43) Consider the following statements regarding
so that it gets passed: amendment to the Constitution:
a) 273 1. All provisions in the Indian Constitution require a
b) 356 special majority in the parliament for its
c) 356 with ratification of half of the state. amendment.
d) 400 with ratification by 3/4th of the state. 2. Federal provisions that require ratification by the
states do not find mention in the Constitution.
Q.40) Consider the following statements: Select the correct answer from the codes given below:
1. 103rd Constitutional Amendment Act made a) 1 only
changes in Articles 14, 15 and 16 of the b) 2 only
Constitution. c) Both 1 and 2
2. 86th Constitutional Amendment Act added Article d) Neither 1 nor 2
21A to the Constitution.
Select the correct answer from the codes given below:
a) 1 only Q.44) Consider the following provisions regarding
b) 2 only amendment to the Constitution:
c) Both 1 and 2 1. A bill seeking a Constitutional amendment can be
d) Neither 1 nor 2 introduced by only a minister and not a private
member.
2. It requires prior approval by the President before
Q.41) Consider the following statements: the introduction of the Constitutional Amendment
1. 86th Constitutional Amendment Act made changes Bill.
in the fundamental rights only. 3. A provision of joint sitting exists in case of the
2. 44th Constitutional Amendment Act dealt with ordinary bills and money bills but not in
promoting equal justice and providing free legal constitutional amendment bills.
aid to the poor. Select the correct answer from the codes given below:
a) 1 and 2 only

TEST 1 6
Contact us : info@onlyias.com

OnlyIAS Nothing Else Visit : dpp.onlyias.in


Contact : +91-7007 931 912

b) 2 and 3 only 4. Goods and Services Tax council


c) 3 only 5. Central Information commission
d) None of the above Which among the above bodies have been inserted to
the Constitution by way of its amendment?
a) 1, 2 and 4 only
Q.45) Consider the following statements: b) 2,3 and 4 only
1. 101st Constitutional Amendment Act imposed c) 3, 4 and 5 only
concurrent power of taxation on the Centre and d) 2, 3, 4 and 5
the states.
2. Goods and Services Tax council was created by an
order of the President. Q.49) Consider the following statements:
3. An amendment to the Goods and Services Tax 1. The Constitution of India is the legal document that
Council requires a Constitutional amendment is devoid of a political philosophy.
under Article 368. 2. Indian Constitution gives precedence to liberalism
Select the correct answer using the codes given over social justice.
below: Select the incorrect answer from the codes given
a) 1 and 3 only below:
b) 2 and 3 only a) 1 only
c) 1, 2 and 3 b) 2 only
d) None of the above c) Both 1 and 2
d) Neither 1 nor 2

Q.46) Which of the following is least appropriate


regarding the amenability of the Constitution? Q.50) Consider the following pairs:
a) For a constitution to be a living document, it must Constitutional Amendment Related
be unfrozen and alterable. Schedule
b) Rigidity is accorded to the provisions that are 1. 91st Constitutional Amendment Act: Tenth
central to the Constitution. Schedule
c) Amenability calls for changes in the rights and 2. 71st Constitutional Amendment Act: Eighth
powers of the federal units in a flexible manner. Schedule
d) Sovereignty of elected representatives of 3. 97th Constitutional Amendment Act: second
Parliament is the basis of amendment procedure. Schedule
4. 89th Constitutional Amendment Act: Sixth
Schedule
Q.47) Consider the following statements: Which of the above pairs is/are correctly matched?
1. For making any changes in use of official language a) 1 and 3 only
requires a special majority of the Parliament along b) 2 and 3 only
with ratification by half of the states. c) 1 and 2 only
2. Amendment to the provisions regarding the use of d) 3 and 4 only
English language in the Parliament requires a
simple majority of the Houses. Q.51) Consider the following statements:
Select the correct answer using the codes given 1. Rearrangement of items amongst the three lists in
below: the seventh schedule of the Constitution does not
a) 1 only require Constitution amendment.
b) 2 only 2. The State Finance Commission was added to the
c) Both 1 and 2 Constitution by way of amendment to the
d) Neither 1 nor 2 Constitution.
Select the correct answer using the codes given
below:
Q.48) Consider the following bodies: a) 1 only
1. NITI Aayog b) 2 only
2. Special Officer for linguistic minorities c) Both 1 and 2
3. National Commission for Backward classes d) Neither 1 nor 2

TEST 1 7
Contact us : info@onlyias.com

OnlyIAS Nothing Else Visit : dpp.onlyias.in


Contact : +91-7007 931 912

4. Appointment of Governor by center: Government


Q.52) Consider the following statements: of India Act,1935
1. Right to Property was removed from the list of Which of the above are incorrectly matched?
Fundamental rights by the 42nd Constitutional a) 1,2 and 4 only
Amendment. b) 2 and 3 only
2. Changes in Article 300A of the Constitution require c) 1 and 4 only
Constitution amendment under Article 368. d) 2,3 and 4 only
Select the correct answer using the codes given
below: Q.56) Consider the following statements in context
a) 1 only of Indian Constitution
b) 2 only 1. The term liberty means license to do what one
c) Both 1 and 2 likes
d) Neither 1 nor 2 2. The term equality means the absence of special
privileges to any section of the society
3. The principles of liberty, equality and fraternity
Q.53) Which of the following statements is most should be treated separately and cannot unified.
appropriate regarding the Constitution of India: Select the incorrect answer from code given below:
a) The Constitution is a document that provides for a) 1 and 2 only
the accumulation of all power only in one b) 2 only
institution. c) 1 and 3 only
b) Enjoyment of Fundamental rights is not subjected d) 1,2 and 3
to the fulfilment of the duties in India.
c) Free and Fair elections provide for the
unprecedented and direct role of the citizens in Q.57) India is a Republic. What does this statement
policy making and its administration. mean?
d) Removal of Judges by the Parliament fixes judicial 1. India can either acquire a foreign territory
accountability to the Parliament. 2. vesting of political sovereignty in the people
3. the absence of any privileged class
Select the correct answer from the code given below
Q.54) Consider the following statements about a) 1 only
features of Indian Constitution: b) 2 and 3 only
1. The Constitution contains the fundamental c) 1 and 3 only
principles of Governance and also d) 2 only
detailed administrative provisions to administer
the nation.
2. The political part of the Constitution has been Q.58) Consider the following statement about
largely drawn from the Canadian Constitution dyarchy
3. Indian Constitution adopted universal adult 1. It is the scheme of governance introduced by the
franchise as a basis of elections to the Lok Sabha Morley-Minto reform
and the state legislative Assemblies ever since the 2. It distinguishes between commercial and political
enactment of Constitution functions of the company.
Select the correct statement/s from the code given 3. Government of India act 1935 recommended for
below: adoption of dyarchy at centre.
a) 1 and 3 only Which of the statement/s given above is/are
b) 2 and 3 only correct?
c) 3 only a) 1 and 2 only
d) 1 only b) 1 and 3 only
c) 3 only
d) 2 and 3 only
Q.55) Consider the following features
1. Procedure for amendment: South Africa
2. Judiciary : French
3. Emergency Provisions: Canada Q.59) Consider the following statements:

TEST 1 8
Contact us : info@onlyias.com

OnlyIAS Nothing Else Visit : dpp.onlyias.in


Contact : +91-7007 931 912

1. Indian council act 1861 made limited and indirect 1. Scope of Judicial review power of the Supreme
provision for election to fill some non-official court of India is wider than the court in the United
seats. States.
2. Separate electorate was extended to Sikhs, Indian 2. American constitution provided for due process of
Christians, Anglo-Indians and Europeans by the law against the procedure established by law in
Government of India Act 1919. Indian Constitution.
3. The Government of India Act 1935 introduced 3. Both India and USA Constitution provides for a
responsible government in the province. single system of court which enforces both the
Which of the statement/s given above is/are central as well as State law.
correct? Which of the statement/s given above is/are
a) 1 and 2 only incorrect?
b) 2 and 3 only a) 1 only
c) 1 and 3 only b) 1 and 2 only
d) 1, 2 and 3 c) 2 and 3 only
d) 1 and 3 only

Q.63) Consider the following statement:


Q.60) Consider the following statements about the
1. Indian Constitution provides a negative concept
Government of India Act 1858:
Secularism which gives equal respect to all
1. The Act introduced system of double government
religions.
in India
2. Western concept of Secularism is positive concept
2. It created a new office of Secretary of State for
which emphasis on complete separation between
India.
the religion and the state
3. The Act made the Governor-General of Bengal as
Select the correct answer from the code given below
the Governor-General of India.
a) 1 Only
Select the correct answer from the code given below:
b) 2 Only
a) 1 and 2 only
c) 1 and 2
b) 2 only
d) Neither 1 nor 2
c) 2 and 3 only
d) 1 and 3 only
Q.64) Consider the following statements:
1. Second Schedule of India Constitution provides
for emoluments, allowances and privileges for
Q.61) Consider the following Statements about Prime Minister of India
Charter Act 1853: 2. The Third Schedule of the Constitution contains
1. It ended the activities of the East India company the oath and affirmation of the President of India.
as a commercial body and made it purely an 3. The fourth schedule of Indian constitution is
administrative body. related to the allocation of seats on the Rajya
2. The Act separated the legislative and executive sabha to the state and the union territories.
function of the Governor-General’s council. Select the correct answer from the code given below
3. The Act introduced, for the first time, local a) 1 and 2 only
representation in Indian (Central) legislative b) 1 and 3 only
council. c) 2 and 3 only
Select the correct answer from the code given below: d) 3 only
a) 1 and 2 only
b) 1 and 3 only
c) 2 and 3 only
Q.65) Consider the following statements about 42nd
d) 1, 2 and 3
constitutional amendment act:
1. The Act made provision for establishment of
administrative tribunals.
Q.62) Consider the following Statements: 2. The Act did away with the requirement of quorum
in the parliament and the state legislatures.

TEST 1 9
Contact us : info@onlyias.com

OnlyIAS Nothing Else Visit : dpp.onlyias.in


Contact : +91-7007 931 912

3. It deleted the right to property from the list of Q.69) Consider the following statements:
Fundamental Rights and made it only a legal right. 1. Indian Independence Act 1947 made constitution
Select the correct answer from the code given below assembly a fully sovereign body.
a) 1 and 2 only 2. With the commencement of constitution of
b) 1 and 3 only India The abolition of Privy Council Jurisdiction
c) 2 and 3 only Act was repealed.
d) 1, 2 and 3 3. The Constitution assembly enacted the preamble
at the first meeting of the assembly so that it can
Q.66) Consider the following statements: act as guiding principle for it.
1. Combination of social and political justice is Select the correct answer from the code given below
known as distributive justice. a) 1 only
2. The ideal of justice- social, political and economic b) 1 and 2 only
in Indian constitution has been taken from French c) 1 and 3 only
Revolution. d) 2 and 3 only
Select the correct answer from the code given below
a) 1 only
b) 2 only
Q.70) With reference to Off Budget Borrowings in
c) 1 and 2
India, consider the following statements:
d) Neither 1 nor 2
1. It is counted in the calculation of the fiscal deficit.
2. Off-budget borrowings are loans that are
taken by the Centre directly.
Q.67) Consider the following statements: 3. This route of financing puts major sources of
1. Preamble is neither a source of power to the funds outside the control of Parliament.
legislature nor part of the constitution. Which of the statements given above is/are correct?
2. Provisions of preamble are not enforceable in a) 1 and 2 only
courts of law. b) 2 and 3 only
3. Preamble cannot be amended as it is not part of c) 3 only
the Constitution of India. d) 1, 2 and 3
Select the correct answer from the code given below
a) 1 and 2 only Q.71) With reference to Pathurghat Peasant uprising,
b) 2 only consider the following statements:
c) 1 and 3 only 1. Increase in agricultural land tax reportedly by 70-
d) 2 and 3 only 80 per cent, by the British government was the
major cause of this uprising.
2. This uprising took place in western parts of India.
3. 3.Peasants used democratic methods to settle the
Q.68) Consider the following statements about the
dispute.
Constitution assembly :
Which of the statements given above is/are correct?
1. It was directly elected by the people of India on
a) 1 only
the basis of the adult franchise.
b) 2 and 3 only
2. The Constitution assembly has representation
c) 1 and 3 only
from all sections of Indian society.
d) 1, 2 and 3
3. The representatives of each community were
elected by the members of that community in the
Q.72) With reference to different models of PPP,
provincial legislative assembly
consider the following pairs:
Select the correct answer from the code given below
Model Feature
a) 1 and 2 only
1. LDO- In this, the governments allow private
b) 1 and 3 only
promoters to rehabilitate and operate a facility
c) 2 and 3 only
during a concession period
d) 1, 2 and 3
2. ROT- the government retains ownership of the
newly created infrastructure

TEST 1 10
Contact us : info@onlyias.com

OnlyIAS Nothing Else Visit : dpp.onlyias.in


Contact : +91-7007 931 912

3. DBFO- In this model, the private party assumes


the entire responsibility
Which of the pairs given above is/are correctly Q.76) He was an Indian social reformer and freedom
matched? fighter from Kerala. He took part in anti-
a) 1 and 2 only untouchability agitations and advocated opening
b) 2 only temples for people of all castes, also participated in
c) 3 only the Vaikom Satyagraha. He is also known for his
d) 1, 2 and 3 founding of the Nair Service Society (NSS).
The above given description refers to which of the
Q.73) With reference to Law Commission of India, following famous personalities of Indian freedom
consider the following statements: struggle?
1. Law Commission of India is a non-statutory body. a) Shri. Balakrishnan Nair
2. The commission is established for a fixed tenure. b) K. Kelappan
3. The first Law Commission was established by the c) Sri Mannathu Padmanabhan
Charter Act of 1833, it was presided by Lord d) E. V. Ramasamy
Macaulay.
Which of the statements given above is/are correct? Q.77) Servants of the People Society, was founded
a) 1 and 2 only by which of the following personalities?
b) 2 and 3 only a) Gopal Krishna Gokhale
c) 1 and 3 only b) Lala Lajpat Rai
d) 1, 2 and 3 c) Surendranath Banarjee
d) Aurobindo Ghosh

Q.78) Consider the statements about NGOs in India


Q.74) With reference to Open Skies treaty, consider 1. Constitutionally NGOs are backed by Article 19
the following statements: 2. NGOs are regulated by FCRA (Foreign contribution
1. This treaty is not related to civil-aviation open Regulation Act)2010 under ministry of Finance
skies agreements. 3. The Constitution Under Union list mentions
2. Under the treaty, a member state can spy on any charitable institutions
part of the host nation. Which of the above statement/s is/are correct?
3. India is not a member of this treaty. a) 1 and 2
Which of the statements given above is/are correct? b) Only 1
a) 1 and 2 only c) 2 and 3
b) 2 and 3 only d) Only 2
c) 1 and 3 only
d) 1, 2 and 3 Q.79) Tadoba Andhari Tiger Reserves, recently in
news is located in
Q.75) With reference to International Energy Agency a) Karnataka
(IEA), consider the following statements: b) Maharashtra
1. It is a Paris-based autonomous intergovernmental c) Chhattisgarh
organisation established in the framework of the d) Telangana
Organisation for Economic Co-operation and
Development (OECD) in 1974. Q.80) Consider the following statements with
2. To be a member, a candidate country must be a reference to G7(Group of seven) countries
member country of the OECD and all OECD 1. Canada, Japan, Italy, UK, USA, France, Germany
members are IEA members. are the 7 members of this group
3. It is best known for the publication of its annual
2. The aggregate GDP G7 countries make is more
World Energy Outlook report. than 50% of world GDP
Which of the statements given above is/are correct? 3. There are no formal criteria for membership, but
a) 1 and 2 only all the members are developed countries.
b) 2 and 3 only 4. Russia was also the part of group but got
c) 1 and 3 only suspended following its annexation of Crimea
d) 1, 2 and 3 Which of the above statement/s is/are correct?
TEST 1 11
Contact us : info@onlyias.com

OnlyIAS Nothing Else Visit : dpp.onlyias.in


Contact : +91-7007 931 912

a) 1,2 and 3 Q.85) The word ‘Leang Tedongnge’ recently news is


b) 2,3 and 4 related to
c) 1,3 and 4 a) Discovery of oldest known speaking language in
d) All of the above East Asia
b) Discovery of oldest known rock painting drawn by
Q.81) India Innovation Index recently seen in news is man
released by c) Discovery of oldest known volcano series in world
a) Department of Science & Technology d) None of the Above
b) NITI Aayog
c) Ministry of ministry of skill development and Q.86) Consider the following statements
entrepreneurship 1. He started newspaper named as “Swaraj” and
d) Pratham NGO authored a book “The Indian Struggle”
2. He was inspired by teachings of Bhagwat Gita and
Q.82) “Prime editing” sometimes seen in news is Swami Vivekananda
related to, 3. In his young age he became the leader of INCs
a) Film editing technology by Amazon radical wing
b) 3D printing editing technology 4. He also served as President of All India Trade
c) Gene editing technology Union Congress (AITUC)
d) Research on Covid Vaccine Above statements are elaborating which of the
following Freedom fighter
Q.83) With reference to Arctic Council, consider the a) Lala Lajpat Rai
following statements b) S. C. Bose
a) It is Intergovernmental council for Arctic area c) Chittaranjan Das
cooperation d) Bal gangadhar Tilak
b) India became member of the council in 2013
c) It was set up with the twin mandate of
environmental protection and sustainable
development Q.87) With reference to ‘Judicial Activism’, consider
d) The Arctic Council cannot enforce its guidelines or the following statements
recommendations. 1. Judicial review
Which of the following statement/s is/are correct? 2. Article 13, 32 and 224
a) A, B and C 3. Supervisory Power of higher courts on lower
b) B, C and D courts
c) A, C and D 4. Public Interest Litigation (PIL)
d) All of the above Which of the above provisions promotes judiciary to
take proactive role to follow ‘Judicial Activism’
Q.84) Consider the following statements with a) 1,2 and 3
respect to NBFC b) 2,3 and 4
1. NBFCs do not include any institution whose c) 1,3 and 4
principal business is that of agriculture activity or d) All of the above
industrial activity.
2. RBI issues licenses of NBFCs and also regulate
them Q.88) Consider the following countries about
3. NBFCs do form a part of the payment and ‘Abraham accord’ recently seen in news
settlement system but can not issue cheques 1. Israel
4. NBFCs are run and regulated under Companies 2. United Arab Emirates (UAE)
Act,1956 3. Bahrain
Which of the following statement/s is/are correct 4. USA
a) Only 1 and 3 Which of the following countries are signatories of
b) Only 1 and 4 this accord?
c) Only 1, 2 and 4 a) 1,2 and 3
d) All of the above b) 2,3 and 4

TEST 1 12
Contact us : info@onlyias.com

OnlyIAS Nothing Else Visit : dpp.onlyias.in


Contact : +91-7007 931 912

c) 1,3 and 4 1. Improving Maternal Health


d) All of the above 2. Combating human immunodeficiency virus (HIV)
3. Financial contributions to the Chief Ministers’
Relief Fund
4. Donations to Prime Minister's Citizen Assistance
Q.89) Consider the following statements and Relief in Emergency Situations Funds
1. It is the "longest running" monthly English (PMCARES) fund
magazine of the country 5. Relief and funds for the welfare of Other
2. It is the magazine run by Ramakrishna Mission Backward Classes
3. Founded at Madras in 1896 at the behest of Select the correct answer using the code given
Swami Vivekanand below:
4. Luminaries like Netaji Subhas Chandra Bose, Bal a) 1, 2 and 4 only
Gangadhar Tilak, Sister Nivedita,Sri Aurobindo, b) 1, 3 and 4 only
SarvepalliRadhakrishnan etc.wrote in the journal c) 1, 2, 4 and 5 only
Find the which of the following magazine described d) 2, 3, 4 and 5 only
by above lines
a) Udbodhana Journal
b) Nav jeevan Journal
c) Prabudhha Bharat Journal Q.93) If you were to travel from the Mollem National
d) Som prakash Journal Park to the Shoolpaneshwar Wildlife Sanctuary,
what is the minimum number of states you would
Q.90) Which of the following are applications of travel through, including the origin and the
quantum computing? destination ?
1. Cyber security and Cryptography a) 3
2. Traffic Optimisation b) 4
3. Weather forecasting c) 5
4. Financial modelling d) 6
5. Drug development
Select the correct answer using the code given
below:
Q.94) According to the E-Waste (Management)
a) 1, 2 and 4 only
Rules, 2016, a “refurbisher” of goods is someone
b) 3, 4 and 5 only
who is engaged in
c) 1, 2, 3 and 5 only
a) Breaking down used items into raw materials for
d) All of the above
manufacturing the same products again.
b) Repairing used electrical equipment to extend its
Q.91) Consider the following pairs:
working life for its originally intended use.
Space Mission/Vehicle Purpose
c) Transforming by-products, waste materials or
1. Hayabusa-2 Interstellar
unwanted products into new materials.
travel to Kupier’s belt
d) None of the Above
2. Chang’e 5 Sample
collection from Moon
3. Ingenuity Vertical
Hovering on Mars Q.95) Consider the following pairs:
Which of the above pairs are correctly matched? Active Volcanoes in News
a) 1 and 2 only Country
b) 2 and 3 only 1. Sangay
c) 1 and 3 only Peru
d) All of the Above 2. Sinabung
Indonesia
Q.92) Which of the following activities can be 3. Semeru
performed by a company to accomplish its Corporate Brunei
Social Responsibility obligations under the Which of the above pairs are incorrectly matched?
Companies Act, 2013? a) 1 only

TEST 1 13
Contact us : info@onlyias.com

OnlyIAS Nothing Else Visit : dpp.onlyias.in


Contact : +91-7007 931 912

b) 1 and 2 only Q.100) The “One Health” approach recognizes that


c) 2 and 3 only the health of people is closely connected to the
d) 1 and 3 only health of animals. In this regard, which of the
following are zoonotic diseases?
1. Salmonella infection
2. Ebola
Q.96) The “Competitiveness and Resilience (CoRe) 3. Distemper Disease
Partnership” Agreement, which seeks to invest in the 4. Lyme disease
development of next-generation mobile networks 5. West Nile Virus Disease
and is strategically important from the perspective of Choose the correct answer using the code given
the Quad grouping, has been signed between Japan below:
and a) 1,2 and 3 only
a) United States of America b) 1,2,4 and 5 only
b) India c) 3,4 and 5 only
c) Australia d) 1,2,3 and 5 only
d) ASEAN

Q.97) With reference to the Attorney General (AG) of


India, consider the following statements:
1. According to the constitution of India, the AG is
appointed by the president by a warrant under his
hand and seal.
2. According to the constitution of India, the AG
shall receive such remuneration as the President
may determine.
3. According to the The Contempt of Courts Act,
1971 , the AG’s consent is required for initiation
of all contempt proceedings.
Which of the above statements is/are correct ?
a) 1 and 2 only
b) 2 only
c) 1 and 3 only
d) 1,2 and 3

Q.98) Which of the following adopted the


“Additional Protocol to the Convention on
Cybercrime” which requires action against racist and
xenophobic material through computer systems?
a) The African Union
b) The Council of Europe
c) The United Nations General Assembly
d) The European union

Q.99) The term “Compulsory Licensing”, is


sometimes seen in the news in the context of
a) Electricity Distribution
b) Data Localization Norms
c) E-Commerce Regulations
d) Drug Manufacturing

TEST 1 14
Contact us : info@onlyias.com

OnlyIAS Nothing Else Visit : dpp.onlyias.in


Contact : +91-7007 931 912

Q.1) Ans: D minorities, backward and tribal areas and


depressed and other backward classes.
Exp:
• Dr. Rajendra Prasad was unanimously elected the
permanent President of the Assembly on 11th Q.4) Ans: A
December 1946 under temporary chairmanship of
Dr. Sachchidananda Sinha. Exp:
• The Objective Resolution was adopted on January • Statement 1 is incorrect: The Preamble provides
22, 1947. liberty of thought, expression, belief, faith and
• The National Flag was adopted on July 22, 1947. worship.
• Ratification of India’s membership of the • Statement 2 is correct: The term ‘liberty’ means
Commonwealth was done in May, 1949. the absence of restraints on the activities of an
individual. It does not mean the absence of laws
as liberty cannot be achieved without laws as
Q.2) Ans: A laws would establish rule of law in the country.
• Statement 3 is correct: Liberty provides
Exp: opportunities for the development of an
• Statement 1 is correct: The Indian Independence individual personality.
Act, 1947 empowered the Constituent Assembly • Statement 4 is correct: Liberty is essential for
to legislate till the new Constitution is drafted and successful functioning of democracy as it would
enforced. No Act of the British Parliament passed not place restrictions on an individual in terms of
after August 15, 1947 was to extend to Dominion his/her innovations.
of India, unless it was extended thereto by a law
of the legislature.
• Statement 2 is incorrect: The dual task of making Q.5) Ans: A
the Constitution and legislation continued till
November 26, 1949 (the date of adoption of the Exp:
Constitution), when the task of making the • Option A is incorrect: In a Republic, the head of
Constitution was over. the State is always either directly or indirectly
• Statement 3 is correct: On January 24, 1950, the elected for a fixed period. In India, the President is
Constituent Assembly held its final session. It did indirectly elected for a term of five years.
not end and continued as the provisional • Option B is correct: The term Republic signifies
parliament of India from January 26, 1950 till the that the political sovereignty is vested in the
formation of new Parliament after the first people of the country.
general elections in 1951-52. • Option C is correct: The term Republic also
signifies that the absence of any privileged class
and hence all public offices are opened to every
Q.3) Ans: D citizen without any discrimination.
Reference: Laxmikanth
Exp:
• Statement 1 is incorrect: The Objective
Resolution resolved to proclaim India as an Q.6) Ans: D
Independent Sovereign Republic. Even the original
Preamble did not contain the words ‘Socialist’ and Exp:
‘Secular’ which were added by 42nd Constitutional • Option 1 is correct: Article 17 provides for
Amendment Act, 1976. abolition of untouchability and thus aims to
• Statement 2 is correct: The Objective Resolution remove social disabilities imposed on certain
proclaimed that “all power and authority of the sections of the society.
Sovereign Independent India, its constituent parts • Option 2 is correct: Article 38 directs the State to
and organs of the Government will be derived secure a social order for the promotion of welfare
from the people of India”. of the people and thus promotes social justice.
• Statement 3 is correct: The Objective Resolution • Option 3 is correct: Article 39A directs the State
proclaimed to provide adequate safeguards for to promote equal justice and to provide free legal
aid to the poor.

TEST 1 15
Contact us : info@onlyias.com

OnlyIAS Nothing Else Visit : dpp.onlyias.in


Contact : +91-7007 931 912

• Option 4 is correct: Article 42 provides for just thus promotes secularism in India. Other Articles
and humane conditions of work and maternity of the Constitution that promote secularism are
relief. Article 14, 25, 26, 27, 28.
• Option 5 is incorrect: Article 49 directs the State
to protect monuments, places and objects of
artistic or historic interest which are declared to Q.9) Ans: D
be of national importance. Thus, it is not related
Exp:
to social justice, rather it is a liberal-intellectual
Statement D is correct:
principle.
• Sarkaria Commission was set up in 1983.
• Option 6 is incorrect: Article 50 directs the State
• Lowering of voting age from 21 to 18 years
to separate the judiciary from the executive in the
occurred in the 61st Constitutional Amendment
public services of the State. Thus, it is a liberal-
Act, 1989 for both the Parliament and the State
intellectual principle.
Legislative Assembly elections.
• 73rd Constitutional Amendment Act, 1992
Q.7) Ans: B accorded the Constitutional status and protection
to the Panchayati Raj Institutions.
Exp: • Kihoto Hollohon case or the Anti defection case
• Statement 1 is correct: The Preamble has been occurred in 1993.
amended only once so far by 42nd Constitutional Hence, the correct sequence is 4-1-2-3.
Amendment Act, 1976. At that time the Prime
Minister of India was Mrs. Indira Gandhi.
• Statement 2 is incorrect: The Supreme Court in Q.10) Ans: D
the Kesavananda Bharati case held that the
Exp:
Preamble is part of the Constitution.
• Statement 1 is incorrect: Constitutional
• Statement 3 is incorrect: The original Preamble
Amendment Bills can be introduced in both the Lok
did not include the words ‘Secular’, ‘Socialist’ and
sabha or the Rajya Sabha. Hence, both Lok Sabha
‘Integrity’ which were added by the 42nd
and Rajya sabha have equal powers with respect to
Constitutional Amendment Act.
the constitutional amendment bills.
• Statement 2 is incorrect: Elections to the
Q.8) Ans: D Parliament and the State Legislature require a
simple majority of the Parliament for its
Exp: amendment. Hence, it lies outside the purview of
• Statement 1 is incorrect: The term ‘Secular’ was Article 368 of the Constitution. Provisions that
added through the 42nd Constitutional require special majority of the Parliament and
Amendment Act, 1976. ratification by half of the States (with simple
• Statement 2 is incorrect: Indian model of majority) include:
secularism is called the soft model of secularism. o Article 368 itself.
It provides for a porous wall of separation o Election of the President and the manner of
between State and religion, i.e. state can election.
intervene in religious matters to give progressive o Extent of executive powers of the Centre and
voices within every religion and to allow States.
disagreement with some aspects of religion, e.g. o Distribution of legislative powers between the
abolition of Sati, Hindu Code Bill, Untouchability Centre and the States.
etc. However, religion is strictly prohibited in state o GST Council
matters, hence disallowing mobilising of electoral o Seventh Schedule
support on religious lines. On the other hand, the o Representation of States in the Parliament.
French model of secularism is called the hard o Supreme Court and High Courts.
model of secularism that provides for strict • Statement 3 is incorrect: There is no time limit
separation of State and religion. prescribed in the Constitution for the ratification
• Statement 3 is correct: Article 15 and 16 prohibits by the states.
State from discrimination on the ground religion
and Article 44 provides for uniform civil code and
TEST 1 16
Contact us : info@onlyias.com

OnlyIAS Nothing Else Visit : dpp.onlyias.in


Contact : +91-7007 931 912

• Statement 4 is correct: Changes in Salaries and • Statement 3 is correct: It abolished the office of
Allowances of the MPs amounts to a Constitutional Secretary of state and transferred his functions to
Amendment outside Article 368. It is done by the the office of secretary of state for commonwealth
simple majority of the Parliament, like any other affairs.
ordinary legislation. The main provisions of the Indian Independence Act,
1947 are as follows:
o It ended the British rule and made India into a
sovereign and independent state as on August
Q.11) Ans: C
15, 1947.
Exp: o Caused the partition of India into two
• Statement 1 is incorrect: Indirect provision for dominions of India and Pakistan, with the right
elections was introduced in the Indian Councils Act, to secede from the British Commonwealth.
1892. The Act provided for the nomination of non- o Abolished the office of Viceroy and provided
official members for both the Central and the for the Governor General for each dominion,
provincial legislative councils. In the Central who was to be appointed by the British King.
Legislative Council, such members were He was the nominal head to act on the advice
nominated on the recommendation of provincial of the Dominion cabinet.
legislative councils and the Bengal chamber of o Abolished the office of the Secretary of State
commerce. In the provincial legislative councils, for India.
such members were nominated on the o Ended the British paramountcy over the
recommendations of district boards, princely states
municipalities, trade associations, Zamindars, o Empowered the Constituent Assembly of the
universities, etc. two dominions to legislate for themselves till
• Statement 2 is correct: Direct elections were the adoption of the new Constitution.
introduced for the first time by the Government of o Abolished the right of the British monarch to
India Act, 1919. It allowed franchise on a certain veto bills. Governor General, in the name of His
criterion of tax, property and education. majesty, had full powers to give assent to the
• Statement 3 is correct: Government of India Act, bills.
1919 granted Franchise to a limited number of
people based on property, tax, education Q.13) Ans: B
• Statement 4 is incorrect: Franchise to 10 percent
of the population was extended as per the Exp:
Government of India Act, 1935. • Statement 1 is incorrect: The Charter Act of 1793
• Statement 5 is correct: Article 326 of the provided for the payment of members of the Board
Constitution states “Elections to the House of the of Control charged to the company. Pitt’s India Act,
People and to the Legislative Assemblies of States 1784 was instrumental as it provided for the
to be on the basis of adult suffrage. The elections creation of the Board of Control to look after the
to the House of the People and to the Legislative civil and military affairs. It separated the
Assembly of every State shall be on the basis of commercial functions from the administrative
adult suffrage”. functions.
• Statement 2 is correct: The Charter Act of 1813
empowered the local government to levy taxes.
Q.12) Ans: B Other provisions of the Charter Act of 1813:
Exp: o Ended the monopoly of the company to trade
with India. However, it still retained its
• Statement 1 is incorrect: It abolished the office of
Viceroy and provided for the Governor General for monopoly over tea trade and trade with China.
each dominion, who was to be appointed by the o Allowed Christian missionaries to propagate
British King. It abolished the right of the British their religion.
o A sum of one lakh rupees was set aside for the
monarch to veto bills.
promotion of education.
• Statement 2 is correct: Governor General, in the
name of His majesty, had full powers to give assent • Statement 3 is incorrect: An all-white seven-
member commission was established under the
to the bills.
TEST 1 17
Contact us : info@onlyias.com

OnlyIAS Nothing Else Visit : dpp.onlyias.in


Contact : +91-7007 931 912

chairmanship of Sir John Simon to report into the the entire territorial area possessed by the British
working of Government of India Act, 1919. It in India.
submitted its report in 1930. The commission • Statement 2 is Incorrect: It ended the activities of
recommended the abolition of dyarchy at the the East India Company as a commercial body,
provinces, establishment of All India federation of which became a purely administrative body.
provinces and princely states, continuation of
communal electorate.
• Statement 4 is incorrect: Ramsay MacDonald, Q.16) Ans: B
British Prime Minister came with the Communal
Award in 1932 which extended the system of Exp:
separate electorate for the depressed classes. • Statement 1 is Incorrect: The Constituent
Mahatma Gandhi undertook fast unto death in Assembly was formed in November 1946 under
opposition to the communal award and this finally the scheme formulated by the Cabinet Mission
culminated in the signing of the Poona Pact Plan.
between the Mahatma Gandhi and Dr. B.R. • Statement 2 is Correct: The Constituent Assembly
Ambedkar. However, Lord Minto was known as was to be a partly elected and partly nominated
the father of communal electorate after he came body.Seats allocated to British provinces were to
up with the separate electorate for the Muslims in be divided among the three principal
the Government of India Act, 1909. communities–Muslims, Sikhs and General (all
except Muslims and Sikhs), in proportion to their
population.
Q.14) Ans: B • Statement 3 is Incorrect: The total strength of the
Constituent Assembly was to be 389. Of these,
Exp: 296 seats were given to British India and 93 seats
The Indian Independence Act 1947 ended the British to the princely states. Out of 296 seats in British
rule in India and declared India as an independent and India, 292 members were to be drawn from the
sovereign state from August 15, 1947. eleven governors’ provinces and four from the
Statement 1 is Correct: designated the Governor- four Chief Commissioners’ provinces.
general of India and provincial governor as
constitutional (nominal) heads of the states. They
were made to act on advice of respective council of Q.17) Ans: A
ministers in all matters
Statement 2 is Incorrect: It provided for the Exp:
governance of each of the dominions and the • Statement 1 is incorrect- The Indian concept of
provinces by the Government of India Act of 1935, till Secularism is different from the western concept
the new Constitutions were framed. The dominions of secularism. In the West, the State is separate
were however authorised to make modifications in from the functioning of all religious institutions
the Act. and groups. Whereas, in India means that the
Statement 3 is Correct: The Assembly also became a state is neutral to all religious groups but not
legislative body. In other words, two separate necessarily separate. The Indian Constitution
functions were assigned to the Assembly, that is, embodies a very positive concept of Secularism.
making of the Constitution for free India and enacting Giving equal respect to all religions equally is one
ordinary laws for the country. of the core components of Indian Constitution.
• Statement 2 is incorrect- The word ‘Secular’ was
added in the Preamble of India by the 42nd
Q.15) Ans: A Amendment Act of 1976 and was not a part of the
original Constitution.
Exp:
• Statement 3 is correct- Equality of opportunity for
• Statement 1 is Correct: It made the Governor-
all citizens in matters of public employment-
General of Bengal as the Governor General of
Article 16- reveals the Secular character of the
India and vested in him all civil and military
Indian State. Article 16 says that no citizen shall,
powers. Thus, the act created, for the first time,
on grounds only of religion, race, caste, sex,
the Government of India having authority over

TEST 1 18
Contact us : info@onlyias.com

OnlyIAS Nothing Else Visit : dpp.onlyias.in


Contact : +91-7007 931 912

descent, place of birth or any of them, be • Statement 3 is correct- The Emergency Provisions
ineligible for any office under the State. in the Constitution in Part XVIII of the Constitution
of India, from Article 352 to 360 converts the
federal structure into a unitary one without the
formal amendment of the Constitution. The
Q.18) Ans: D rationality behind the incorporation is to
safeguard the sovereignty, unity, integrity and
Exp:
security of the country, the democratic political
• Statement 1 is correct- Universal Adult Franchise
system and the Constitution.
establishes political equality among all the adults.
It ensures that all the people are having hands in
the electing ruler and hence the government is Q.20) Ans: B
democratic. It also makes democracy broad-
based. Exp:
• Statement 2 is incorrect- Universal franchise/ • Pair 1 is incorrect-Fourth Schedule Deals with the
suffrage gives the right to vote to every citizen Allocation of seats in the Rajya Sabha (the Upper
who is 18 years of age or above, regardless of House of Parliament) to the State and Union
wealth, income, gender, social status, race, Territory.
ethnicity etc. The 61st Amendment Act lowered • Pair 2 is correct- Third schedule deals with Forms
the voting age from 21 to 18 years. of oaths and Affirmations for-
• Statement 3 is correct- It opens up new hopes o Union Ministers of India
and vistas for weaker sections of the society as it o Parliament Election Candidates
empowers people to elect their own governments o Members of Parliament (MPs)
in a democratic manner. The Political equality o Supreme Court Judges
established by the Universal Adult Franchise o Comptroller and Auditor General
provides a sense of empowerment to all o State Ministers
especially the weaker sections of the society. o State Legislature Elections’ Candidates
o State Legislature Members
o High Court Judges
Q.19) Ans: D • Pair 3 is correct-. Eight schedule deals with the
Languages recognised in the Constitution. There
Exp:
are 22 official languages recognized by the Indian
• Statement 1 is correct- The Indian constitution
Constitution- Assamese, Bengali, Bodo, Dogri
provides for an Integrated judicial system. The
(Dongri), Gujarati, Hindi,Kannada, Kashmiri,
Supreme court is at the top below it there are
Konkani, Mathili(Maithili) ,Malayalam, Manipuri,
high courts at state. Under a high court, there is a
Marathi, Nepali, Oriya, Punjabi, Sanskrit, Santhali,
hierarchy of subordinate courts i.e., district and
Sindhi, Tamil, Telugu, Urdu.
other lower courts. This single system of courts
• Pair 4 is incorrect- Ninth schedule deals with the
enforces both the central and state laws.
state acts and regulations that deal with land
• Statement 2 is correct- The American constitution
reforms and abolition of the zamindari system. It
provides for the “due process of law” while the
also deals with the acts and regulations of the
Indian constitution provides for the “procedure
Parliament dealing with other matters.
established by law”. The “Due process of law”
gives wider scope to the Supreme Court to grant
protection to the rights of its citizens. It can Q.21) Ans: B
declare laws violating rights rights void not only
on substantive grounds of being unlawful, but also Exp:
on procedural grounds of being unreasonable. On • Statement 1 is incorrect- Kesavananda Bharati
other hand our supreme court while determining case of 1973- The Supreme court of India ruled
constitutionality of law examines only whether that the constituent power of Parliament under
law is within the power of authority concerned or article 368(Article 368 in Part XX of the
not. Constitution deals with the power of parliament
to amend the constitution and its procedures)

TEST 1 19
Contact us : info@onlyias.com

OnlyIAS Nothing Else Visit : dpp.onlyias.in


Contact : +91-7007 931 912

does not enable it to alter the basic structure of Q.23) Ans: B


the constitution. However, the constitution is
neither too rigid nor too flexible in terms of Exp:
amendment. • Statement 1 is correct: Some provisions of the
• Statement 2 is correct - Some provisions of the Charter Act, 1793:
Constitution can be amended by a simple majority o Renewal of Company’s privileges for another
of the parliament. These amendments do not term of 20 years.
come under Article 368. These provisions include: o Debarring senior officials of the Company from
o Admission or establishment of new states, leaving India without permission.
Formation of new states and alteration of o Separation of revenue administration from the
areas, boundaries or names of existing states. judicial functions. Abolition of Maal Adalats.
o Privileges of the Parliament, its members and o Payment of members of the board of control
its committees. from the revenues of India.
o Abolition or creation of legislative councils in • Statement 2 is incorrect: It was the Charter Act of
states. 1833 which was termed as the St. Helena Act.
o Conferment of more jurisdiction on the • Statement 3 is incorrect: The strength of the Court
Supreme Court. of Directors was not reduced by the Charter Act of
o Citizenship-acquisition and termination. 1793.
o Allowances, privileges and so on of the
president, the governors, the Speakers,
Q.24) Ans: C
judges, etc.
o Quorum in Parliament. Exp:
o Salaries and allowances of the members of • Statement 1 is incorrect: The Montagu Chelmsford
Parliament. reforms or the Government of India Act, 1919
o Rules of procedure in Parliament. introduced bicameralism at the central level.
o Number of puisne judges in the Supreme • Statement 2 is incorrect: Ordinance making power
Court. was extended to the viceroy by the Indian Councils
o Elections to Parliament and state legislatures. Act, 1861.
o Delimitation of constituencies. • Statement 3 is correct: The elected members were
o Union territories indirectly elected by the members of the electoral
• Statement 3 is incorrect: Use of the English college. Local bodies elected the members of the
language in Parliament requires a simple majority electoral college, who further elected the
of parliament for amendment and this members of the provincial legislatures who in turn
amendment does not come under Article 368. elected the members of the central legislature.
• Statement 4 is incorrect: Formation of Muslim
League occurred in December 1905 by Nawab
Q.22) Ans: D Salimullah of Dacca, Nawab Mohsin-ul-Mulk and
Exp: Waqar-ul- Mulk. Hence, Muslim League preceded
• Statement 1 is incorrect: The dual system of the Act of 1909.
government was introduced by the Pitt’s India Act,
1784 where the Court of Directors of the Company Q.25) Ans: B
were to look after the commercial affairs of the
Company while the Board of Control was Exp:
responsible for civil, military and revenue functions • Statement 1 is incorrect: The Charter Act of 1833
of the Company. made efforts for the abolition of slavery in India.
• Statement 2 is incorrect: The Act of 1786 was Other provisions of the Charter Act of 1833 are:
enacted to extend special privileges to Lord o Governor General of Bengal became the
Cornwallis. He was made both the governor Governor General of India (Lord William
general and the commander-in-chief. Later this Bentinck was the first such Governor
power was extended to all the future governor General of India) with all the civil and
generals. military powers.

TEST 1 20
Contact us : info@onlyias.com

OnlyIAS Nothing Else Visit : dpp.onlyias.in


Contact : +91-7007 931 912

o It completely ended the monopoly of the o Extended the principle of Communal


Company as a commercial body which electorate for depressed classes, women, and
became a purely administrative entity. labour.
• Statement 2 is correct: It was during the tenure of • Statement 2 is incorrect: Sir Elijah Impey was the
Lord Ellenborough that slavery was abolished from first chief justice of the Supreme Court established
India in 1843. under the Regulating Act of 1773 while Sir Maurice
Gwyer was the first chief Justice of the Federal
Court established as per the Government of India
Act, 1935.
Q.26) Ans: D

Exp: Q.28) Ans: C


• Statement 1 is incorrect: The office of the
secretary of state was created in the Government Exp:
of India Act, 1858. Hence, it was not created before • Option C is correct:
the revolt of 1857. o The direct elections were introduced by the
• Statement 2 is incorrect: Board of Control and the Government of India Act, 1919 or the Montagu
Court of Directors which provided for the dual Chelmsford reforms.
system of government in the Regulating Act of o The Board of Control was abolished by the
1773 was abolished by the Government of India Government of India Act, 1858 and was
Act,1858. Hence, the Secretary of State was not the established by the Pitt’s India Act of 1784.
head of the Board of Control. He was a member of o Separate Electorates for the Women was
the British cabinet and oversaw the entire Indian introduced by the Government of India Act,
administration. 1935.
• Statement 3 is incorrect: The office of the o The provision for the establishment of the
Secretary of State was abolished in the Indian Federal Court was provided in the Government
Independence Act of 1947. The council of India of India Act, 1935. The Court was formally
which was established in the Government of India established in 1937.
Act, 1858 to assist the Secretary of state was o The Vernacular Press Act was passed by Lord
abolished in the Government of India Act, 1935. Lytton in 1878. It was called the ‘gagging act’
for imposing severe restrictions on Indian
language newspapers.
Q.27) Ans: A o The report of the Simon Commission was laid
in 1930.
Exp:
Hence, the correct sequence is 2-5-1-6-3-4.
• Statement 1 is correct: The Federal Court was
created by the Government of India Act, 1935.
Other provisions of this Act are: Q.29) Ans: B
o The Act provided for the establishment of the
Reserve Bank of India. Exp:
o It divided the powers between the Federal and • Statement 1 is incorrect: The Government of India
the provincial subjects on two lists- Federal list Act, 1935 provided for dyarchy at the central level.
and the Provincial list, with all the residuary The subjects at the Federal level were divided into
powers given to the viceroy. reserved and transferred subjects. Dyarchy in the
o Burma was separated from India as per the provinces was abolished in this Act.
Government of India Act, 1935. • Statement 2 is correct: The Government of India
o Provided for the establishment of the Federal Act, 1935 abolished the Council of India
Court, which was set up in 1937. established by Government of India act 1858 which
o Provided for the establishment of Federal was established to assist the secretary of state.This
Public Service Commission, Provincial Public act provides a team of advisors to the secretary of
Service Commission and Joint Public Service state.
Commission.
o Extended Franchise to 10 percent of the
population. Q.30) Ans: B

TEST 1 21
Contact us : info@onlyias.com

OnlyIAS Nothing Else Visit : dpp.onlyias.in


Contact : +91-7007 931 912

Exp: six to fourteen years and thus provides for social


• Statement 1 is incorrect: The division of subjects justice.
into the three lists were provided for in the
Government of India Act, 1935. All the residuary
powers were under the control of the Governor Q.33) Ans: D
General.
Exp:
• Statement 2 is correct: The provision for
• Statement 1 is incorrect: The Constituent
establishment of a federal public service
Assembly was constituted as per the provisions of
commission, provincial public service commission
the Cabinet Mission Plan. The Cabinet Mission was
and joint public service commission for two or
announced in 1946. Its members included Pethick
more provinces was given by the Government of
Lawrence, Stafford Cripps, and A.V. Alexander.
India, 1935.
• Statement 2 is incorrect: The Indian Independence
Act, 1947 or the Mountbatten plan provided for
the partition of India and created two independent
Q.31) Ans: B dominions of India and Pakistan. It empowered the
Constituent Assembly of the two dominions to
Exp: adopt any constitution as it thinks fit and to repeal
• Statement 1 is incorrect: National Flag was any act of the British Parliament, including the
adopted by the Constituent Assembly on the 22nd Indian Independence Act, 1947 itself. The main
July 1947. Hence, it was adopted before provisions of the Indian Independence Act, 1947
independence. are as follows:
• Statement 2 is correct: The Ninth Schedule was o It ended the British rule and made India into a
inserted as a result of the 1st Constitutional sovereign and independent state as on August
Amendment in 1951. It was added to shield the 15, 1947.
laws placed in from Judicial Review. o Caused the partition of India into two
• Statement 3 is incorrect: The idea of Concurrent dominions of India and Pakistan, with the right
list in the Indian Constitution has been borrowed to secede from the British Commonwealth.
from the Constitution of Australia. o Abolished the office of Viceroy and provided
• Statement 4 is incorrect: The discussions on the for the Governor General for each dominion,
Round Table Conferences led to the enactment of who was to be appointed by the British King.
the Government of India Act, 1935. The Poona Pact He was the nominal head to act on the advice
was signed in 1930 between Mahatma Gandhi and of the Dominion cabinet.
Dr. B.R. Ambedkar after the announcement of the o Abolished the office of the Secretary of State
Communal Award. for India.
o Ended the British paramountcy over the
princely states
Q.32) Ans: D o Empowered the Constituent Assembly of the
Exp: two dominions to legislate for themselves till
• Pair 1 is correct: Article 16 provides for equality of
the adoption of the new Constitution.
opportunity in matters of public employment. o Abolished the right of the British monarch to
• Pair 2 is correct: Article 25 provides for freedom
veto bills. Governor General, in the name of His
of conscience and free profession, practice and majesty, had full powers to give assent to the
propagation of religion. The right to free bills.
profession provides an individual to declare
his/her religious beliefs and faith openly and
freely. Q.34) Ans: B
• Pair 3 is correct: Article 17 provides for abolition
of untouchability and thus protects the dignity of Exp:
an individual. • Statement 1 is correct: The amending act of 1781
• Pair 4 is correct: Article 21A provides for free and was passed to rectify the errors of the Regulating
compulsory education to all children of the age of Act of 1773. Hence, it was also known as the Act of
Settlement.
TEST 1 22
Contact us : info@onlyias.com

OnlyIAS Nothing Else Visit : dpp.onlyias.in


Contact : +91-7007 931 912

• Statement 2 is Correct: The outlined the Q.37) Ans: A


jurisdiction of the Supreme Court at Calcutta which
was established by the Regulating Act of 1773. It Exp:
held that its jurisdiction was extended to all the • Statement 1 is Correct: The portfolio system was
inhabitants of Calcutta. The Hindu subjects were to recognised by the Indian Council Act, 1861. It was
be tried as per the Hindu laws while the Muslims introduced by Lord Canning in 1859 before the
subjects were to be tried as per the Mohammedan Indian Council Act, 1861.
laws. • Statement 2 is incorrect: Lee Commission (1923-
• Statement 3 is incorrect: It held that the appeals 24) dealt with the reforms of the civil services. It
from the Provincial courts were to be taken to the recommended the establishment of the Public
Governor-General-in council and not to the Service Commission.
supreme court. The Governor-General-in Council
was empowered to frame regulations for the
Provincial courts and the Council. Q.38) Ans: A

Exp:
Q.35) Ans: B • Statement 1 is correct: 52nd Constitutional
Amendment Act, 1985 added Tenth Schedule to
Exp:
the Constitution. It deals with the provisions
• Statement 1 is incorrect: It was the Charter Act of
related to the Anti-defection law.
1833 that completely abolished the commercial
• Statement 2 is incorrect: 93rd Constitutional
activities of the Company. The charter Act of 1813
Amendment Act, 2005 empowered the state to
ended its trade monopoly except for trade in tea
make special provisions for the socially and
and with China. The Charter Act of 1833 purely
educationally backward classes/ SCs/STs in
transformed it into an administrative entity.
educational institutions including the private
• Statement 2 is correct: The Charter Act of 1853
educational institutions. However, minority
introduced local representation into the Indian
educational institutions are exempted from these
Legislative Council. Four members were appointed
provisions. While the 86th Constitutional
to the Indian Legislative Council by the provincial
Amendment Act, 2002 dealt with the Article 21A of
governments of Bombay, Madras, Bengal and
the Constitution which aims to provide free and
Agra.
compulsory education to all the children of the age
of 6-14 years.
Q.36) Ans: B

Exp: Q.39) Ans: B


• Statement 1 is incorrect: The Charter Act of 1793
Exp:
did not introduce the Dual system of Government.
• Statement B is correct: The total sanctioned
It was introduced by the Pitt’s India Act, 1784
strength of the Lok Sabha is 552. The present 17th
which provided for the establishment of the Board
Lok Sabha has 545 members. On the given day,
of Control to look after the civil, military and
only 541 are present and 8 abstain from voting.
revenue affairs of the Company and the Court of
Hence, the total members that are present and
Directors to look after the commercial affairs of the
voting are 533. The bill that is taken up voting is the
Company.
Constitutional Amendment Bill. A bill seeking to
• Statement 2 is correct: The Charter Act of 1853
amend the provisions like Fundamental Rights and
introduced a system of open competition for the
Directive Principles of State Policy requires a
recruitment of the civil servants. Accordingly, the
special majority of the Parliament. Article 50 deals
Macaulay Committee on the Indian Civil Services
with the separation of judiciary from the executive.
was appointed in 1854.
Hence it requires a special majority in the
• Statement 3 is correct: The Government of India
Parliament. Special majority involves. A) Majority
Act, 1935 abolished Dyarchy from the provinces
of the total membership of the House, here it is
and introduced complete provincial autonomy
545/2 i.e., 273 and, B) majority of the two-thirds of
instead.
the members present and voting, here it is 2/3 of

TEST 1 23
Contact us : info@onlyias.com

OnlyIAS Nothing Else Visit : dpp.onlyias.in


Contact : +91-7007 931 912

533 i.e., 356. Hence, it must be passed by 356 income, status, facilities, and opportunities. The
members. provision regarding promotion of equal justice and
providing free legal aid to the poor is contained in
Article 39A of the Constitution. It was inserted by
Q.40) Ans: B the 42nd Constitutional Amendment Act, 1976.
Exp:
• Statement 1 is incorrect: 103rd Constitutional
Amendment Act of 2019 seeks to provide Q.42) Ans: B
reservation to the Economically Weaker sections in
the Educational Institutions and in Public Exp:
Employment. It amended the Article 15 and 16 of • Statement 1 is incorrect: As per Article 368 of the
the Constitution and made no change in the Article Constitution, a Constitutional Amendment Bill can
14. be introduced in either house of the Parliament,
o Article 15 deals with prohibition on not necessarily in the Lok Sabha. Rajya Sabha has
discrimination on certain grounds. It ensures equal powers with Lok Sabha in case of
that states do not discriminate against any Constitutional Amendment Bills.
citizen on the grounds only of religion, race, • Statement 2 is correct: Constitutional Amendment
caste, sex, or place of birth. bills cannot be introduced in the state legislatures.
o Article 16 deals with equality of opportunity The state legislatures can only pass a resolution
for all citizens in the matters related to public requesting the Parliament for the creation or
employment. abolition of the state legislative council which can
• Statement 2 is correct: Article 21A declares that also be rejected or accepted by the Parliament.
State shall provide free and compulsory education Hence, state legislatures cannot initiate a
to all children of the age of six and fourteen years. constitutional amendment.
It was added by the 86th Constitutional • Statement 3 is incorrect: The President must
Amendment Act, 2002. To fulfil the objectives of mandatorily give his assent to the Constitution
Article 21A, the Parliament enacted the Right of Amendment bill. He can neither withhold assent
Children to Free and Compulsory Education (RTE) nor return the bill. This provision was added by the
Act, 2009. 24th Constitutional Amendment Act, 1971.
• Statement 4 is correct: There are certain
provisions in the Constitution that seeks to amend
the Federal provisions of the constitution. Such
Q.41) Ans: D provisions require special majority in the
Parliament and require ratification by half of the
Exp:
state legislatures with a simple majority. These
• Statement 1 is incorrect: 86th Constitutional
provisions are:
Amendment Act, 2002 made changes in the
o Power to amend the constitution (Article 368)
Fundamental rights, Directive principles of the
itself.
State Policy and Fundamental duties. It inserted
o Election of the President and the manner of
Article 21A to the chapter on Fundamental rights.
election.
Article 21A declares that State shall provide free
o Extent of executive powers of the Centre and
and compulsory education to all children of the age
States.
of six and fourteen years. It also made changes to
o Distribution of legislative powers between the
Article 45. Article 45 deals with providing early
Centre and the States.
childhood care and education for all children until
o GST Council
they attain the age of six years.It also inserted
o Seventh Schedule
additional fundamental duty in article 51A for
o Representation of States in the Parliament.
parents or the guardian to provide opportunity of
o Supreme Court and High Courts.
education to his child or ward between the age of
6 to 14 years.
• Statement 2 is incorrect: 44th Constitutional
Amendment Act, 1978 inserted a directive Q.43) Ans: D
principle that aims to minimize inequalities in
TEST 1 24
Contact us : info@onlyias.com

OnlyIAS Nothing Else Visit : dpp.onlyias.in


Contact : +91-7007 931 912

Exp: • Statement 1 is correct: 101st Constitutional


• Statement 1 is incorrect: some provisions in Amendment Act deals with the concurrent power
the Constitution can be amended by the simple of taxation by the Centre and the states. Both
majority in the Parliament in the manner like Parliament and State legislatures have concurrent
the passage of ordinary legislation which are powers of taxation in matters related to Goods and
outside the scope of article 368 of constitution. Services tax.
Further certain provisions require a special • Statement 2 is correct: 101st Constitutional
majority along with ratification by half of the Amendment Act, 2016 inserted Article 279¬A in
states. the Constitution. Article 279A empowers the
• Statement 2 is incorrect: Federal provisions President to create a Goods and Services Tax
that require ratification by half of the state Council or the GST Council with the powers to
legislatures with a simple majority are make decisions in matters related to GST.
mentioned in the Constitution under Article • Statement 3 is correct: An Amendment to the
368. These provisions are: provisions relating to GST council requires
o Power to amend the constitution (Article constitutional amendment under Article 368 of the
368) itself. Constitution. It requires a special majority in the
o Election of the President and the manner Parliament and ratification by half of the state
of election. legislatures by simple majority.
o Extent of executive powers of the Centre
and States.
o Distribution of legislative powers between
the Centre and the States. Q.46) Ans: C
o GST Council Exp:
o Seventh Schedule • Statement 1 is correct: Constitution makers kept in
o Representation of States in the Parliament. mind that changing times would require
o Supreme Court and High Courts. amendment to the constitution. Hence, to make it
a living document that is coherent with its times,
they kept the option open for its amendment in
Q.44) Ans: D future. Hence, our constitution is not a frozen
document. It is subject to alterations and
Exp: modifications with changing times.
• Statement 1 is incorrect: A Constitutional • Statement 2 is correct: Provisions like
Amendment bill can be introduced by both a Fundamental rights, DPSPs, several federal
minister as well as a private member (member of provisions, the process of amendment is rigid. It
the House who is not a minister). requires a special majority in the Parliament while
• Statement 2 is incorrect: The introduction of the some even require ratification by half of the state
Constitutional amendment bill does not require legislatures. Hence, amendment to the
prior approval of the President before its constitution is a mix of both rigid and flexible
introduction. provisions.
• Statement 3 is incorrect: A provision of joint sitting • Statement 3 is incorrect: Our Constitution
is provided in case of the ordinary bills. However, envisages a federal polity and hence provisions
when a deadlock occurs between the two houses that deal with the rights and powers of the states
in case of passage of money bill or the cannot be amended in a very flexible manner like
constitutional amendment bill, there is no the ordinary legislative process of the Parliament.
provision of a joint sitting provided in the • Statement 4 is correct: As only elected
Constitution. representatives are empowered to consider and
take final decision on question of amendment.
Thus sovereignty of elected representatives is the
basis of amendment procedure.
Q.45) Ans: C Source: NCERT
Exp:

TEST 1 25
Contact us : info@onlyias.com

OnlyIAS Nothing Else Visit : dpp.onlyias.in


Contact : +91-7007 931 912

Q.47) Ans: B • Statement 5 is incorrect: Central Information


Commission is not a Constitutional body. It was
Exp: constituted in 2005 by the Gazette notification
• Statement 1 is incorrect:For makinging any under the Right to Information Act, 2005.
changes in use of official language, a simple
majority is required.which is outside the scope of
Article 368.
Provision which requires special majority of Q.49) Ans: C
parliament and also the consent of half of the state
with simple majority are as follows: Exp:
o Election of the President and the manner of • Statement 1 is incorrect: The Constitution of India
election. not only is a legal document that outlines the laws
o Extent of executive powers of the Centre and that would govern the country but also at the same
States. time it adheres to certain political philosophies as
o Distribution of legislative powers between the well like those of freedom, equality, social justice,
Centre and the States. etc. Hence, it is a mixture of the two.
o GST Council • Statement 2 is incorrect: Unlike the western
o Seventh Schedule concept of liberalism, in Indian constitution it is
o Representation of States in the Parliament. linked to social justice and community values. The
o Supreme Court and High Courts. reservation of Scheduled Castes and Scheduled
• Statement 2 is correct: Amendment to the Tribes is the testimony that while granting
provisions regarding use of English language equality, the Constitution also seeks to provide
requires a simple majority of the Parliament. This affirmative action in favor of the backward classes,
amendment falls outside the purview of who have been a victim of historical injustice.
amendment under Article 368 of the Constitution.

Q.50) Ans: C
Q.48) Ans: B Exp:
Exp: • Statement 1 is correct: 91st Constitutional
• Statement 1 is incorrect: NITI Aayog is neither a Amendment Act dealt with the strengthening of
Constitutional nor a statutory body. It was the Anti-defection law. According to this, the total
established in 2015 as the think tank of the nation number of ministers, including the Prime Minister,
by an executive resolution of the Government of shall not be more than 15% of the total strength of
India. the Lok Sabha.
• Statement 2 is correct: The original Constitution • Statement 2 is correct: 71st Constitutional
did not contain any provision regarding the special Amendment Act added Konkani, Manipuri, and
officer for linguistic minorities. It was added to the Nepali to the Eighth Schedule of the Constitution.
constitution by the 7th Constitutional Amendment • Statement 3 is incorrect: 97th Constitutional
Act, 1956 and added to Part XVII of the Amendment Act gave constitutional status to the
Constitution under Article 350-B. It falls under the Cooperative societies by inserting part IX-B to the
ministry of minority affairs. Constitution of the country.
• Statement 3 is correct: National Commission for • Statement 4 is incorrect: 89th Constitutional
Backward classes was added to the Constitution of Amendment Act divided the National Commission
India by way of the 102nd Constitutional of Scheduled Castes and the National Commission
Amendment Act, 2018. It is mentioned in the on Scheduled Tribes into separate bodies. Sixth
Article 338-B of the Constitution of India. Schedule of the constitution deals with the
• Statement 4 is correct: Goods and Service Tax administration of the tribal areas in the state of
council was added by the 101st Constitutional Assam, Meghalaya, Tripura, and Mizoram.
Amendment Act, 2016 which inserted Article 279-
A. Q.51) Ans: B

TEST 1 26
Contact us : info@onlyias.com

OnlyIAS Nothing Else Visit : dpp.onlyias.in


Contact : +91-7007 931 912

Exp: the enforcement of these duties. Hence, it does


• Statement 1 is incorrect: Rearrangement of items not make the enjoyment of rights conditional upon
amongst the three lists require amendment of the the fulfilment of the duties.
Constitution with the special majority of the • Statement c is incorrect: Conduction of free and
Parliament and with ratification by half of the state fair elections is essential for the functioning of a
legislatures with a simple majority. The 42nd democracy. However, in the Indian case, citizens
Constitutional Amendment Act, 1976 is an have a limited role in making policy decisions as
example here. It transferred five subjects Indian Constitution envisages indirect democracy
(education, forests, weights and measures, and not direct. It would be unsuitable to have
protection of wild animals and birds and direct democracy for a vast diverse country like
administration of justice) from the Concurrent list India. Hence, elected representatives play an
to the state list. important role in day-to-day policy decisions and
• Statement 2 is correct: The provision related to the running of administration.
State Finance Commission was added by the 73rd • Statement d is incorrect: Judges are removed by
and 74th Constitutional Amendment Act which the special majority of the two houses of the
added provisions regarding the Panchayats and the Parliament on the grounds of proved misbehaviour
Municipalities. Articles 243I and the 243Y deals and incapacity. However, to ensure the
with the constitution of the finance commission. independence of the Judiciary they are not
accountable to the Parliament but to the
Constitution itself.
Q.52) Ans: D

Exp:
Q.54) Ans: A
• Statement 1 is incorrect: The Right to Property was
removed from the list of fundamental rights by
removing articles 19(1)(f) and 31 of the Exp:
• Statement 1 is correct: Those matters which in
Constitution by the 44th Constitutional
Amendment Act, 1978. Presently, it is placed under other modern democratic countries have been
Article 300A of the Constitution. left to ordinary legislation or established political
• Statement 2 is incorrect: Article 300A of the
conventions have also been included in the
Constitution contains provisions Right to Property constitution itself in India. The Constitution of
which reads that no person shall be deprived of his India thus not only contains the fundamental
property except by authority of law. Hence, it does principles of governance but also detailed
not form part of the Fundamental rights or the administrative provisions.
• Statement 2 is incorrect: The political part of the
provisions that require amendment under 368 of
the Constitution. Constitution i.e the principle of Cabinet
Government and the relations between the
Executive and the Legislature have been largely
drawn from the British Constitution. This was
Q.53) Ans: B largely because of familiarity with the system
prevailing during colonial times.
Exp: • Statement 3 is correct: The Indian Constitution
• Statement a is incorrect: The Constitution does adopted universal adult franchise as a basis of
not favour the monopoly or the accumulation of elections to the Lok Sabha and the state
power under one institution. It rather promotes legislative assemblies ever since enactment of the
fragmentation of power among the various organs Constitution. The voting age was fixed at 21 years
of government like the Legislature, executive and when the constitution was enacted. The 61st
the Judiciary. The Constitution favours the system Constitutional Amendment Act of 1988 provided
of internal checks and balances amongst these that Every citizen who is not less than 18 years of
institutions. age got a right to vote without any discrimination
• Statement b is correct: The 42nd Constitutional of caste, race, religion, sex, literacy, wealth and so
Amendment Act, 1976 inserted Fundamental on.
duties under Article 51A in Part IV-A of the
Constitution. However, the Constitution is silent on
TEST 1 27
Contact us : info@onlyias.com

OnlyIAS Nothing Else Visit : dpp.onlyias.in


Contact : +91-7007 931 912

Q.55) Ans: D the few over the many. Equality without liberty,
would kill individual initiative.
Exp:
Important provisions borrowed from Government of
India Act, 1935 are Federal Scheme, Office of Q.57) Ans: B
governor, Judiciary, Public Service Commissions,
Emergency provisions and administrative details. Exp:
• Statement 1 is incorrect: The word sovereign
• Option 1 is correct: Procedure for
amendment of the constitution has been implies that India is neither a dependency nor a
derived from the South African constitution. dominion of any other nation, but an independent
The election of member of Rajya Sabha is also state. There is no authority or state above it, and
derived from South African constitution it is free to conduct its own affairs (both internal
• Option 2 is incorrect: The office of judiciary
and external). Being a sovereign state, India can
has been derived from the Government of also acquire a foreign territory or cede a part of its
India Act,1935. territory in favour of a foreign state.
• Statement 2 is correct: The term Republic in our
• Option 3 is incorrect: The Emergency
provisions of Indian Constitution are Preamble indicates that India has a president as
borrowed from the Government of India an elected head. He is elected indirectly for a
Act,1935. fixed period of five years. A republic also means
• Option 4 is incorrect: The office of governor
vesting of political sovereignty in the people and
is borrowed from the Government of India not in a single individual like a king
• Statement 3 is correct: Moreover, it also implies
Act,1935. However, the mode of appointment
of governor by the centre has been taken the absence of any privileged class and hence all
from the Canadian constitution public offices being opened to every citizen
without any discrimination.

Q.56) Ans: C
Q.58) Ans: C
Exp:
Exp:
• Statement 1 is incorrect: The term ‘liberty’ means • Statement 1 is incorrect: Dyarchy is the dual
the absence of restraints on the activities of scheme of governance. In was introduced in the
individuals, and at the same time, providing province by the Montagu-Chelmsford reform
opportunities for the development of individual (Government of India Act 1919). It divided
personalities. liberty does not mean ‘license’ to do provincial subjects into two parts i.e. transferred
what one likes, and has to be enjoyed within the subjects which were administered by the
limitations mentioned in the Constitution itself. governor with the aid of ministers responsible to
• Statement 2 is correct: The term ‘equality’ means the legislative council and reserved subjects which
the absence of special privileges to any section of were administered by the governor and his
the society, and the provision of adequate executive council without being responsible to the
opportunities for all individuals without any legislative council.
discrimination. The Preamble secures to all • Statement 2 is incorrect: Distinguishing between
citizens of India equality of status and commercial and political function of a company
opportunity. was known as Double government system. It was
• Statement 3 is incorrect: The principles of liberty, introduced by the Pitt’s India Act 1784. It allowed
equality and fraternity cannot treat as separate the Court of directors to manage commercial
items in a trinity. They form a union of trinity in affairs and created a new body called the Board of
the sense that to divorce one from the other is to Control to manage Political affairs.
defeat the very purpose of democracy. Equality • Statement 3 is correct: Government of India act
cannot be divorced from liberty, Liberty cannot be 1935 recommended for adoption of dyarchy at
divorced from equality, Nor can liberty and centre. But this provision never came into
equality be divorced from fraternity. Without operation.
equality, liberty would produce the supremacy of

TEST 1 28
Contact us : info@onlyias.com

OnlyIAS Nothing Else Visit : dpp.onlyias.in


Contact : +91-7007 931 912

Q.59) Ans: B company as a commercial body and made it


purely an administrative body. The Act provided
Exp: that the company’s territories in India were held
• Statement 1 is incorrect: It was Indian council act by it in trust for his Majesty, His heirs and
1892 which made limited and indirect provision successors.
for use of elections in filling up some non-official • Statement 2 is Correct: Act separated for the first
seats both in provincial legislative council and time legislative and executive function of the
central level. But the word election was not used Governor-General’s council. Act provided addition
in the act. of six new members called as legislative
• Statement 2 is Correct: Principle of communal councillors to the council. This legislative wing of
representation was extended by providing the council functioned as a mini Parliament.
Separate electorate to Sikhs, Indian Christians, • Statement 3 is Correct: Act introduced for first
Anglo-Indians and Europeans by the Government time local representation in the central legislative
of India Act 1919. council. Out of six new legislative members of
• Statement 3 is Correct: Government of India Act governor-general legislative council, four were
1935 provides for responsible government in appointed by the local governments of Madras,
provinces where the governor is required to act Bombay, Bengal and Agra
with the advice of ministers responsible to the
provincial legislature.

Q.62) Ans: D

Q.60) Ans: B Exp:


• Statement 1 is incorrect and Statement 2 is
Exp: Correct: Scope of Judicial review of the Supreme
• Statement 1 is incorrect: System of double court in India is narrower than the court in the
government was introduced by the Pitt’s India Act United States. Reason behind it is American
1784. Act Distinguished between commercial and constitution provided for due process of law
political function of a company. It allowed the against the procedure established by law in Indian
Court of directors to manage commercial affairs Constitution.
and created a new body called the Board of • Statement 3 is incorrect: In India there is an
Control to manage Political affairs. Government of integrated judicial system where the Supreme
India Act 1858 abolished the system of double Court stands at the top. Below it there is a High
government which was created by the Pitt’s India Court at State level. Under a high court there is a
Act 1784 hierarchy of subordinate courts that is district
• Statement 2 is Correct: Government of India Act court and other lower courts. Hence this single
1858 created the office of Secretary of State for system of courts enforce both central laws as well
India. It has complete authority and control over as state laws. Unlike USA where federal laws are
Indian administration. Secretary of State was a enforced by the federal judiciary and state laws is
member of British cabinet and also ultimately enforced by the state judiciary
responsible to British parliament
• Statement 3 is incorrect: It was the Charter Act of
1833 which made the Governor-General of Bengal Q.63) Ans: D
as the Governor-General of India. Government of
India Act 1858 changed the designation of Exp:
Governor-General of India to Viceroy of India. • Statement 1 is incorrect: Due to multi religious
nature of Indian society westen concept of
secularism which focuses on complete separation
between religion and state is not applicable in
Q.61) Ans: C India. Hence, Indian Constitution provides a
positive concept Secularism which gives equal
Exp:
respect to all religions.
• Statement 1 is incorrect: It was Charter Act 1833
• Statement 2 is incorrect: Western concept of
which ended the activities of the East India
Secularism is negative concept which emphasis on

TEST 1 29
Contact us : info@onlyias.com

OnlyIAS Nothing Else Visit : dpp.onlyias.in


Contact : +91-7007 931 912

complete separation between the religion and the • Statement 1 is incorrect: Combination of the
state social and economic justice is known as
distributive justice. Social justice denotes the
equal treatment of all citizen without any social
Q.64) Ans: D discrimination based on caste, colour, religion,
race, sex and so on. Whereas Economic justice is
Exp:
denoted as non-discrimination between people
• Statement 1 is incorrect: Second schedule of India
on the basic of economic factor.
Constitution provides for emoluments, allowances
• Statement 2 is incorrect: The ideal of justice-
and privileges for President of India, Governor of
social, political and economic in Indian
State, Speaker and Deputy speaker of Lok Sabha
constitution has been taken from Russian
and state legislative assembly, chairman and
Revolution. Ideals of Liberty, equality and
deputy chairman of Rajya Sabha, chairman and
fraternity have been taken from the French
deputy chairman of legislative council of state,
revolution.
Judges of supreme court and high courts,
Comptroller and Auditor General of India. There is
no mention of the Prime minister in it.
• Statement 2 is incorrect: The Third Schedule of Q.67) Ans: B
the Constitution contains the oath and affirmation
of Union and state ministers, candidate for Exp:
election of parliament and state legislature, • Statement 1 is incorrect: Preamble was enacted
member of Parliament and the state legislature, by constitution assembly after the rest of the
judges of supreme court and high court, constitution was already enacted to ensure the
Comptroller and Auditor General of India. There is conformity with the constitution adopted by the
no mention of the President and governor in it. assembly. The Supreme court in the Keshvananda
• Statement 3 is Correct: allocation of seats on the Bharati case held that Preamble is a part of the
Rajya Sabha to the state and the union territories constitution. However, it must be noted that
is mentioned in the fourth schedule of Indian preamble is neither source of power to legislature
constitution nor a prohibition upon the power of legislature
• Statement 2 is Correct: Preamble is non-
justiciable i.e. Provisions of preamble are not
enforceable in courts of law.
Q.65) Ans: A • Statement 3 is incorrect: In Berubari union case
Supreme court upheld that preamble cannot be
Exp:
amended as it is not part of the constitution. But
• Statement 1 is Correct and Statement 2 is
in Kesavananda Bharati case Supreme court
Correct: 42nd constitutional amendment act
changed its opinion and held that the preamble
which is also known as Mini-Constitution added
can be amended subject to the condition that no
part XIV A in which provided for creation of
amendment is done to the “basic feature”.
administrative tribunal and tribunals for other
matters. The Act also did away with the
requirement of quorum in the parliament and the
state legislatures which was later restored by the Q.68) Ans: C
44th constitutional amendment act
• Statement 3 is incorrect: The right to property is Exp:
deleted from the list of Fundamental Rights and • Statement 1 is incorrect: Constitution assembly
made it only a legal right by the 44th was partly elected and partly nominated body.
constitutional amendment act 1978. Members are indirectly elected by the members
of provincial assemblies, who themselves were
elected on a limited franchise which is based on
Q.66) Ans: D tax, property and education. Hence it can be say
that Constitution assembly was not directly
Exp: elected by the people of India on the basis of the
adult franchise

TEST 1 30
Contact us : info@onlyias.com

OnlyIAS Nothing Else Visit : dpp.onlyias.in


Contact : +91-7007 931 912

• Statement 2 is Correct: Though Constitution • Statement 2 is incorrect: Off-budget borrowings


assembly was not directly elected by the people are loans that are taken not by the Centre
of India on the basis of the adult franchise directly, but by another public institution which
assembly has representation of all section of borrows on the directions of the central
Indian society- Hindu, Muslim, Sikhs, parsi, Anglo- government. Such borrowings are used to fulfil
Indians, Indian Christians, SCs, STs including the government’s expenditure needs.
women of all these sections. • Statement 3 is correct: As a result, as a
• Statement 3 is Correct: In the constitution Comptroller and Auditor General report of 2019
assembly the representatives of each community points out, this route of financing puts major
were elected by the members of that community sources of funds outside the control of
in the provincial legislative assembly and voting Parliament. “Such off-budget financing is not part
was done by the method of proportional of the calculation of the fiscal indicators despite
representation by the means of single fiscal implications,” said the report.
transferable vote. NEWS : One of the most sought-after details in any
Union Budget is the level of fiscal deficit. As such, it is
keenly watched by rating agencies — both inside and
outside the country. One of the ways to do this is by
Q.69) Ans: A resorting to “off-budget borrowings”.
Exp:
• Statement 1 is Correct: Indian Independence Act
1947 made constitution assembly a fully Q.71) Ans: C
sovereign body which can frame any constitution
it pleased. Act also empowered assembly to alter Exp:
or abrogate any law made by British Parliament in • Statement 1 is correct: After the British
relation to the India annexation of Assam in 1826, surveys of the vast
• Statement 2 is incorrect: With the lands of the state began. On the basis of such
commencement of constitution of India Indian surveys, the British began to impose land taxes,
Independence Act 1947 and Government of India much to the resentment of the farmers. In 1893,
act 1935 was repealed but the abolition of Privy the British government decided to increase
Council Jurisdiction Act continued to operate. agricultural land tax reportedly by 70- 80 per cent.
• Statement 3 is incorrect: Preamble was enacted “Up until then the peasants would pay taxes in
by constitution assembly after the rest of the kind or provide a service in lieu of cash.
constitution was already enacted to ensure the • Statement 2 is incorrect: This uprising took place
conformity with the constitution adopted by the in Assam.
assembly • Statement 3 is correct: Across Assam, peasants
began protesting the move by organising Raij
Mel’s, or peaceful peoples’ conventions.
“According to Guwahati-based author Arup Kumar
Q.70) Ans: C Dutta, who has written a book — Pothorughat —
on the incident, despite these gatherings being
Exp: democratic, the British perceived them as
• Statement 1 is incorrect: One of the most sought- “breeding grounds for sedition”. “So, whenever
after details in any Union Budget is the level of there was a Raij Mel, the British used to come
fiscal deficit. It is essentially the gap between down on it with a heavy hand to disperse them,”
what the central government spends and what it he said.
earns. In other words, it is the level of borrowings NEWS: Patharughat peasant uprising took place on
by the Union government. One of the ways to do January 28, 1894 in Assam.
this is by resorting to “off-budget borrowings”.
Such borrowings are a way for the Centre to
finance its expenditures while keeping the debt
off the books — so that it is not counted in the Q.72) Ans: C
calculation of fiscal deficit.

TEST 1 31
Contact us : info@onlyias.com

OnlyIAS Nothing Else Visit : dpp.onlyias.in


Contact : +91-7007 931 912

Exp: independent India. The first Law Commission of


Build-Operate-Lease-Transfer (BOLT): In this independent India was established in 1955 for a
approach, the government gives a concession to a three-year term. Since then, twenty one more
private entity to build a facility (and possibly design it Commissions have been established.
as well), own the facility, lease the facility to the NEWS : The Supreme Court has asked the Home and
public sector and then at the end of the lease period Law Ministries to explain the nearly three-year-long
transfer the ownership of the facility to the lapse in making appointments to the Law
government. Commission.

• Pair 1 is incorrectly matched : Lease-Develop-


Operate (LDO): Here, the government or the
public sector entity retains ownership of the Q.74) Ans: D
newly created infrastructure facility and receives
payments in terms of a lease agreement with the Exp:
private promoter. This approach is mostly • Statement 1 is correct: This treaty is not related
followed in the development of airport facilities. to civil-aviation open skies agreements.
• Pair 2 is incorrectly matched : Rehabilitate- • Statement 2 is correct: Under the treaty, a
Operate-Transfer (ROT): Under this approach, the member state can spy on any part of the host
governments/local bodies allow private nation, with the latter’s consent. A country can
promoters to rehabilitate and operate a facility undertake aerial imaging over the host state after
during a concession period. After the concession giving notice 72 hours before and sharing its exact
period, the project is transferred back to flight path 24 hours before.
governments/local bodies. • Statement 3 is correct: Currently, 34 states are
• Pair 3 is correctly matched : DBFO (Design, Build, party to the treaty while a 35th, Kyrgyzstan, has
Finance and Operate): In this model, the private signed but not ratified it. India is not a member of
party assumes the entire responsibility for the this treaty.
design, construction, finance, and operate the
project for the period of concession.

NEWS : Cabinet approves Deepening and


Optimization of Inner Harbour Facilities at Paradip
Port.

Q.73) Ans: D

Exp:
• Statement 1 is correct: Law Commission of India
is an executive body established by an order of
the Government of India. Its major function is to Q.75) Ans: C
work for legal reform.The Law Commission of
India is a non-statutory body constituted by the Exp:
Government of India from time to time. • Statement 1 is correct: The International Energy
• Statement 2 is correct: Its membership primarily Agency is a Paris-based autonomous
comprises legal experts, who are entrusted a intergovernmental organisation established in the
mandate by the Government. The commission is framework of the Organisation for Economic Co-
established for a fixed tenure and works as an operation and Development (OECD) in 1974 in the
advisory body to the Ministry of Law and Justice. wake of the 1973 oil crisis.
• Statement 3 is correct: The first Law Commission • Statement 2 is incorrect: It has 30 members at
was established during the British raj era in 1834 present. The IEA family also includes eight
by the Charter Act of 1833. It was presided by association countries. A candidate country must
Lord Macaulay After that, three more be a member country of the OECD. But all OECD
Commissions were established in pre- members are not IEA members.
TEST 1 32
Contact us : info@onlyias.com

OnlyIAS Nothing Else Visit : dpp.onlyias.in


Contact : +91-7007 931 912

• Statement 3 is correct: It is best known for the • Statement 2 is incorrect: FCRA(Foreign


publication of its annual World Energy Outlook. contribution Regulation Act) 2010 comes under
NEWS : India signs Strategic Partnership Agreement Ministry of Home affairs
with International Energy Agency (IEA). The • Statement 3 is incorrect: The Constitution under
agreement seeks to strengthen mutual trust and Concurrent list mentions charitable institutions
cooperation & enhance global energy security, • NGOs are in the news because the
stability and sustainability. Ministry of Home Affairs (MHA) tightened over
sight on funds received by non-governmental
organisations (NGOs). MHA led out series of
guidelines and charter to make NGOs and Banks
Q.76) Ans: C comply with new provisions of amended
FCRA(Foreign contribution Regulation Act)2010
Exp:
Mannathu Padmanabhan (2 January 1878 – 25
February 1970) was an Indian social reformer and
freedom fighter from the south-western state of Q.79) Ans: B
Kerala. He is recognised as the founder of the •Nair
Service Society (NSS), which claims to represent the Exp:
Nair community that constitutes 12.10% (From KMS • The Tadoba Andhari Tiger Reserve is a wildlife
2011) of the population of the state. Padmanabhan is sanctuary located in Chandrapur district of
considered as a visionary reformer who organised the Maharashtra and it is Maharashtra’s oldest and
Nair community under the NSS. He took part in anti- largest national park. The reserve includes the
untouchability agitations and advocated opening Tadoba National Park and the Andhari Wildlife
temples for people of all castes. He also participated Sanctuary.
in the Vaikom Satyagraha. • Recently in the news because the Central
NEWS : PM pays tribute to Sri Mannathu government had proposed auctioning of Bander
Padmanabhan on his Jayanti. coal mine blocks close to Tadoba Andhari Tiger
Reserve. But recently the coal ministry has
withdrawn Bander mine from the list of 41 coal
Q.77) Ans: B blocks put up for auction as the mine lies in the
eco sensitive zone.
Exp:
Servants of the People Society (SOPS) (Lok Sevak
Mandal) is a non-profit social service organization Q.80) Ans: C
founded by Lala Lajpat Rai, a prominent leader in the
Indian Independence movement, in 1921 in Lahore. Exp:
Lajpat Rai’s most important writings include - The Group of Seven is an intergovernmental
• The Story of My Deportation (1908) organization founded in 1975
• Arya Samaj (1915) • Statement 1 is correct: Canada, Japan, Italy, UK,
• The United States of America: A Hindu’s USA, France, Germany are the 7 members of this
Impression (1916) group. The seven countries involved are also the
• England’s Debt to India: A Historical Narrative of largest IMF-Advanced countries in the world
Britain’s Fiscal Policy in India (1917) • Statement 2 is incorrect: The aggregate GDP G7
countries make around 30% of world GDP
• Statement 3 is correct: There is no formal criteria
Q.78) Ans: B for membership, but while creating the group all
the members were industrial powerhouse and
Exp: developed countries.
• Statement 1 is correct: Constitutionally NGOs are • Statement 4 is correct: Russia was also the part of
backed by Article 19(1)(c) which allows the right group but got suspended following its annexation
to form associations. of Crimea in 2014 G7 was recently in the news
because The United Kingdom has invited the

TEST 1 33
Contact us : info@onlyias.com

OnlyIAS Nothing Else Visit : dpp.onlyias.in


Contact : +91-7007 931 912

Prime Minister of India to attend the G7 summit


to be held in June 2021. Along with India,
Australia and South Korea are also invited.
Simple trick: Statement 2 cannot be correct because
only China and India together contribute around 27%
of world GDP. So, from this you can guess that G7 will
not make 50% of the world's GDP as all other
countries in the world together will definitely make
more than 23%

Q.81) Ans: B

Exp:
• India Innovation Index Report 2020 released by
NITI Aayog. Index modelled on the Global • Statement 1 is correct: It is a high level
Innovation Index (GII). Intergovernmental council for providing
• In hill and North-East states Himachal Pradesh
cooperation,coordination and interaction among
topped the rankings, followed by Uttarakhand, the Arctic States, Arctic indigenous communities
Manipur and Sikkim. on common Arctic issues, in particular on issues of
• In Union territories/Small states Delhi has topped
sustainable development and environmental
the index. protection in the Arctic.
• Statement 2 is incorrect: Ottawa Declaration
declares Canada, the Kingdom of Denmark,
Finland, Iceland, Norway, the Russian Federation,
Sweden and the United States of America as a
member of the Arctic Council. India is not a
member of this council. India got Observer status
in 2013. Observers don't have any voting rights.
• Statement 3 is correct: It is a council set up with
the twin mandate of environmental protection
and sustainable development. Deals with issues
such as the change in biodiversity, melting sea ice,
plastic pollution and black carbon.
• Statement 4 is correct: The Arctic Council does
not and cannot implement or enforce its
Q.82) Ans: C guidelines, assessments or recommendations.
• India launched its first scientific expedition to
Exp: the Arctic Ocean in 2007 and opened a research
• Prime editing is the latest gene-editing base named "Himadri” at the International
technology used to remove/edit a gene in smooth Arctic Research Base at Ny-Alesund, Svalbard,
muscle tissue. Norway in July 2008 for carrying out studies in
• The technology directly writes new genetic disciplines like Glaciology, Atmospheric sciences
information into a targeted DNA site. & Biological sciences.
• It is more easy and efficient than traditional • In the news because the Indian Government has
CRISPR technology. recently released a draft Arctic Policy document in
order to solicit public comments.
• Simple Trick: All the members of this council are
Q.83) Ans: c at the periphery of the Arctic region.

Exp:
Q.84) Ans: B

TEST 1 34
Contact us : info@onlyias.com

OnlyIAS Nothing Else Visit : dpp.onlyias.in


Contact : +91-7007 931 912

Exp:
• Statement 1 is correct: NBFCs do not include any
institution whose principal business is that of
agriculture activity, industrial activity, purchase,
sale of any goods or providing any services and
sale/purchase/construction of immovable
property.
• Statement 2 is not correct: NBFCs get licenses by
RBI but they do not regulate them. NBFCs get
regulated by different agencies based on the role
the individual NBFC play. Example: Housing
Finance institution regulated by national Housing
Bank
• Statement 3 is not correct: NBFCs do NOT form a Q.86) Ans: B
part of the payment and settlement system and
but cannot issue cheques Exp:
• Statement 4 is correct: NBFCs are run and • 23rd January, 2021, India celebrated the 125th
regulated under Companies Act,1956 birth anniversary of Netaji Subhas Chandra Bose
o In news because RBI has proposed a shift in its as ‘Parakram Divas’ annually.
regulatory approach towards India’s non- • Subhash Chandra Bose was twice elected
banking financial companies. (NBFCs) President of the Indian National Congress, (1938-
o Proposed regulations are 1. Creation of 4 Haripur and 1939-Tripuri)
layer regulatory framework 2. classification of • He resigned from the Congress Presidency in 1939
Non-Performing Assets (NPAs) of base layer and organised the All India Forward Bloc, a faction
NBFCs from 180 days to 90 days overdue. within the Congress in Bengal.
• In 1942(Germany), he started ‘Azad Hind Radio’
and established ‘Free India Centre’(precursor of
the Azad Hind Government). He arrived in
Singapore in July 1943 and took charge of the
Indian National Army (INA) or Azad Hind Fauj.

Q.87) Ans: C

Exp:
• Judicial review is the power of the judiciary to
interpret the constitution and to declare any such
law or order of the legislature and executive void,
if it finds them in conflict with the Constitution.
Q.85) Ans: B Hence JR gives scope for Judicial activism by giving
responsibility for interpretation in the hands of
Exp: the judiciary.
• Archaeologist discovered oldest known cave art, a • Judicial Activism means the proactive role played
life-sized painting of wild Pig that was painted by the judiciary in the protection of the rights of
around 45,500 years ago in Sulawesi, Indonesia citizens and in the promotion of justice in the
• The Sulawesi warty pig painting was found in the society.
limestone cave of Leang Tedongnge • Article 13, 32 and 226(and not 224) are provisions
• The previously oldest dated rock painting was at in the constitution itself for the judiciary to adopt
least 43,900 years old, is a depiction of hybrid a proactive role. Article 13 says Laws inconsistent
human-animal beings hunting Sulawesi warty pigs with or in derogation of fundamental rights shall
and dwarf bovids. be void. Article 32 and 226 deals with the types of
writs in Supreme court and High court
respectively.

TEST 1 35
Contact us : info@onlyias.com

OnlyIAS Nothing Else Visit : dpp.onlyias.in


Contact : +91-7007 931 912

• Supervisory Power of higher courts on lower


courts is itself a form of judicial activism
• Public Interest Litigation (PIL) is most popular Q.90) Ans: D
form of Judicial Activism
• Judicial Activism was recently in the news because Exp:
the act of the Supreme Court staying Quantum Computing:
implementation of three farm laws was seen by • Quantum computing is the use of quantum
many as an act of judicial activism/overreach. phenomena such as superposition and
entanglement to perform computation. Quantum
Computers are believed to be able to solve certain
computational problems substantially faster than
Q.88) Ans: A
classical computers. Google has recently made
Exp: headlines proclaiming the achievement of
• Recently, the Abraham Accord signed between quantum supremacy, where its computers can
Israel, the United Arab Emirates and Bahrain, and perform a task that a conventional computer can’t
mediated (NOT signed) by the USA. • Applications
• As per the agreements, the UAE and Bahrain will o Cybersecurity & Cryptography
establish embassies and exchange ambassadors. o Drug Design & Development
Working together with Israel across a range of o Computational Chemistry
sectors, including tourism, trade, healthcare and o Artificial Intelligence & Machine Learning
security. The Abraham Accords also open the o Financial Modelling
door for Muslims around the world to visit the o Logistics and traffic Optimisation
historic sites in Israel and to peacefully pray at o Weather forecasting
Al-Aqsa Mosque in Jerusalem, the third holiest • The Ministry of Electronics and Information
site in Islam. Technology (MeitY) has recently announced a
• Significance of the Accord is very important collaboration with Amazon Web Services (AWS) to
because the agreement shows how the Arab develop a Quantum Computing Applications Lab
countries are gradually decoupling themselves in the country. The lab is meant to provide access
from the Palestine question. to quantum computing development
• Before this deal there were the only two peace environment for the developer, scientific and
deals between Israel and the Arab States, Egypt academic communities
was the first Arab State to sign a peace deal with
Israel in 1979 and 2nd was Jordan signed a peace
pact in 1994.
Q.91) Ans: B

Exp:
Q.89) Ans: C • Option 1 is incorrect: Hayabusa2 is an asteroid
sample-return mission operated by the Japanese
Exp:
state space agency JAXA. Launched in 2014, it
• “Prabuddha Bharat Journal” was recently in news
surveyed the one-kilometre wide Ryugu asteroid
because journal celebrated 125th Anniversary
for a year and half and took samples. It recently
• The journal ‘Prabuddha Bharata’ has been an
returned the samples to earth and its mission has
important medium for spreading the message of
now been extended through at least 2031.
India’s ancient spiritual wisdom.
• Option 2 is correct: Change’s is the fifth lunar
• It is the "longest running" monthly English
exploration mission of the Chinese Lunar
magazine of the country
Exploration Program, and China's first lunar
• It is the magazine run by Ramakrishna Mission
sample-return mission. The space probe carrying
• Prabuddha Bharata was founded in 1896 by P.
rocks and soil from the Moon recently returned to
Aiyasami, B. R. Rajam Iyer, G. G. Narasimhacharya,
the earth safely. This is the first lunar sample
and B. V. Kamesvara Iyer, in Madras at the behest
return since 1976 when the former Soviet Union's
of Swami Vivekananda (Hence statement C is
Luna-24 returned with Lunar material.
incorrect)

TEST 1 36
Contact us : info@onlyias.com

OnlyIAS Nothing Else Visit : dpp.onlyias.in


Contact : +91-7007 931 912

• Option 3 is correct: Ingenuity is a small robotic News: Govt amends CSR rules to boost transparency,
helicopter located on Mars since 18 February flexibility
2021. It successfully completed the first powered Modi govt amends Companies Act, makes PM
controlled flight by an aircraft on a planet besides CARES eligible to receive CSR funds
Earth, taking off vertically, hovering and landing.

Q.92) Ans: C Q.93) Ans: A

Exp: Exp:
• Options 1, 2, 4 and 5 are correct: • Mollem national Park
Recently, section 467 of the Companies Act, 2013 o Recently, A controversy has erupted over
was amended to make the PM CARES fund (Prime three proposed infrastructure projects — a
Minister's Citizen Assistance and Relief in highway expansion, the double-tracking of a
Emergency Situations Funds) eligible to receive railway line, and a power transmission line —
CSR funding from corporates. With this the broad which if initiated could mean the cutting of
categories of programmes stipulated by the over 30,000 trees in the dense evergreen and
government to ensure accountability are: semi-evergreen forests of Mollem at the edge
o Eradicating extreme hunger and poverty of the Western Ghats, 60 km from Goa’s
o Promotion of education capital Panaji. The ‘Save Mollem’ campaign
o Promoting gender equality and empowering has drawn in everyone from celebrities to
women politicians, protesting the lack of assessment
o Reducing child mortality of the threats posed by three proposed
o Improving maternal health infrastructure projects.
o Combating human immunodeficiency virus, o Facts about Mollem national park:
acquired, immune deficiency syndrome, ▪ Located in Goa
malaria and other diseases ▪ It is home to waterfalls, such as
o Ensuring environmental sustainability, Dudhsagar Falls and Tambdi Falls
o Employment enhancing vocational skills, ▪ The parkland is also home to a
social business projects community of nomadic buffalo
o Contribution to the Prime Minister's National herders known as the Dhangar.
Relief Fund or any other fund set up by the ▪ Major Fauna: leopard, barking deer,
Central Government or the State Bengal tiger, bonnet macaque, civet,
Governments for socio-economic flying squirrel, gaur, Malabar giant
development, and squirrel, mouse deer, pangolin,
o Donations to Prime Minister's Citizen porcupine, slender loris, sambar,
Assistance and Relief in Emergency Situations spotted deer, wild boar, etc.
Funds (PMCARES) fund ▪ Threats: surface mining and transport
o Relief and funds for the welfare of the of manganese and iron ores
Scheduled Castes, the Scheduled Tribes, other • The Shoolpaneshwar Wildlife Sanctuary is located
backward classes, minorities and women and in the Narmada district of Gujarat. Recently a
such other matters as may be prescribed. controversy has erupted over centre notification
• Option 3 is not correct: The government has said which classified 121 villages around the
financial contributions to CMs’ relief fund would Shoolpaneshwar wildlife sanctuary as Eco
not be considered as CSR spending. In contrast to sensitive zones. As per the provisions of the
the government’s position, a parliamentary notification, land falling in the eco-sensitive zone
committee on said financial contributions to the cannot be transferred for non-agricultural use for
Chief Ministers’ Relief Fund should also be treated commercial, industrial or residential purposes.
as corporate social responsibility (CSR) spending
and necessary changes should be made to the
Companies Law to make this change.
Q.94) Ans: B

TEST 1 37
Contact us : info@onlyias.com

OnlyIAS Nothing Else Visit : dpp.onlyias.in


Contact : +91-7007 931 912

Exp: Q.95) Ans: D


• Refurbishment of e-waste:
o According to the E-Waste (Management) Exp:
Rules, 2016
▪ 'refurbishment' means repairing of used
electrical and electronic equipment as for
extending its working life for its originally
intended use and selling the same in the
market or returning to owner
▪ A 'refurbisher' for the purpose of these
rules, means any company or undertaking
registered under the Factories Act, 1948
or the Companies Act, 1956 or both or
district industries centre engaged in
refurbishment of used electrical and
electronic equipment
▪ A recycler’ - means any person who is
engaged in recycling and reprocessing of
waste electrical and electronic equipment • Pair 1 is incorrectly matched:
or assemblies or their components. o Recently, Ecuador’s Sangay volcano erupted,
Extra Edge by ONLYIAS spewing clouds of ash as high as 8,500 meters
• Some other important definitions in the E-Waste (about 28, 890 feet) into the sky. The sky of
(Management) Rules, 2016 Riobamba, the capital of the province of
o ‘Orphaned products’ means non-branded Chimborazo, was overcast by an ash cloud.
or assembled electrical and electronic Riobamba is at least 50 kilometers away from
equipment or those produced by a the volcano.
company, which has closed its operations; • Pair 2 is correctly matched and pair 3 is
o Extended Producer Responsibility’ means incorrectly matched:
responsibility of any producer of electrical o Both Sinabung and Semeru are located in
or electronic equipment, for channelisation Indonesia and recently erupted. While Mt.
of e-waste to ensure environmentally Sinabung is located in the sumatra province,
sound management of such waste.It may Semeru is located in East java. It may be noted
comprise of implementing take back that Indonesia sits on the Pacific ring of fire and
system or setting up of collection centres or therefore is tectonically active
both and having agreed arrangements with
authorised dismantler or recycler either NEWS: Ecuador’s Sangay volcano erupts, spews ash 8
individually or collectively through a kilometers | World News,The Indian Express
Producer Responsibility Organisation
recognised by producer.
o ‘Channelisation’ means to direct the path
for movement of e-wastes from collection
onwards to authorised dismantler or
recycler. In case of fluorescent and other
mercury containing lamps, where recyclers
are not available, this means path for
movement from collection centre to
Treatment, Storage and Disposal Facility;
NEWS: Ensure scientific management of e-waste,
environmental crimes as serious as assault: NGT to
CPCB - The Hindu

TEST 1 38
Contact us : info@onlyias.com

OnlyIAS Nothing Else Visit : dpp.onlyias.in


Contact : +91-7007 931 912

constitution, “the president shall appoint a person


who is qualified to be appointed as a judge of the
supreme court to be Attorney General for India.
“Hence the president is not mandated to appoint
AG by a warrant under his hand and seal (unlike
the CAG,the Governor and the Supreme Court and
High Court Judge)
o The same article stipulates that the AG shall
hold office during the pleasure of the
president and shall receive such remuneration
as the President may determine.
• Statement 3 is incorrect: Section 15 of the The
Contempt of Courts Act, 1971, lays down the
procedure on how a case for contempt of court
can be initiated.
o In the case of the Supreme Court, the
Attorney General or the Solicitor General, and
Q.96) Ans: A in the case of High Courts, the Advocate
General, may bring in a motion before the
Exp: court for initiating a case of criminal
• US-Japan ‘CoRe’ Partnership contempt.
o Recently, The US and Japan have pledged o However, if the motion is brought by any
US$4.5 billion for a ‘Competitiveness and other person, the consent in writing of the
Resilience (CoRe) Partnership’ to be invested Attorney General or the Advocate General is
in the research, development and deployment required.
of next-generation mobile networks (‘6G’ or ▪ Thus, The AG’s consent is mandatory
‘Beyond 5G’). when a private citizen wants to
o Crucially, the Competitiveness and Resilience initiate a case of contempt of court
Partnership will increase cooperation in against a person
emerging technologies – an area that has o However, when the court itself initiates a
been typically dominated by China. It reflects contempt of court case, the AG’s consent is
a rapidly evolving effort by the Biden not required. This is because the court is
administration to develop new initiatives and exercising its inherent powers under the
bolster cooperation with allies across the Constitution (Article 129 and Article 215) to
Indo-Pacific, a region that has been identified punish for contempt and such Constitutional
by the US as consequential for the growth of powers cannot be restricted because the AG
high-quality infrastructure that will facilitate declined to grant consent.
the expansion of emerging technologies. ▪ The Supreme Court’s recent
o This comes in the backdrop of Chinese judgment in “Vijay Kurle & Ors, being
ambitions to dominate the development of Suo Motu Contempt Petition
new age technologies such as 5G and (Criminal) No. 2 of 2019” settles the
quantum computing and the recent pledge to position that in suo motu criminal
invest a mammoth $1.4 trillion in emerging contempt cases, the consent of
technologies. Attorney General is not required.
NEWS: The rising sun in India-Japan relations - The NEWS : Explained: Why is consent of A-G required to
Hindu initiate contempt proceedings? | Explained
News,The Indian Express
‘An icon of India’s legal system’: Tributes pour in for
Q.97) Ans: B Soli Sorabjee | India News,The Indian Express

Exp:
• Statement 1 is incorrect and Statement 2 is Q.98) Ans: B
correct: According to article 76 of the
TEST 1 39
Contact us : info@onlyias.com

OnlyIAS Nothing Else Visit : dpp.onlyias.in


Contact : +91-7007 931 912

Exp: for the manufacture of patented drugs in


• Additional Protocol to the Convention on a public health emergency.
Cybercrime: ▪ Section 100 of the Patents Act enables the
o Additional Protocol to the Convention on Central Government to use patented
Cybercrime, concerning the criminalisation of inventions for government purposes.
acts of a racist and xenophobic nature NEWS : Why Centre Not Considering Compulsory
committed through computer systems is a Licensing For COVID Drugs Like Remdesivir,
follow up protocol to the Council of Europe Tocilizumab? Supreme Court Asks (livelaw.in)
Convention on Cybercrime, also called the
BUDAPEST CONVENTION on CYBERCRIMES.
Final text of this protocol was adopted by the
Q.100) Ans: B
Council of Europe Committee of Ministers on
Exp:
7 November 2002.
• Options 1,2,4 and 5 are correct:
o The Protocol requires participating States to
o ONE HEALTH APPROACH
criminalize the dissemination of racist and
▪ One Health is an approach that recognizes
xenophobic material through computer
that the health of people is closely
systems, as well as of racist and xenophobic-
connected to the health of animals and
motivated threats and insults.
our shared environment.
o Two of the English-speaking states in Europe,
▪ The areas of work in which a One Health
Ireland and the United Kingdom, have not
approach is particularly relevant include
signed the additional protocol.
food safety, the control of zoonoses
NEWS: On global cybercrime, India votes in favour of
(diseases that can spread between
Russia-led resolution | India News,The Indian
animals and humans, such as flu, rabies
Express + BLACK LIVES MATTER MOVEMENT (always
and Rift Valley Fever), and combating
in news)
antibiotic resistance (when bacteria
change after being exposed to antibiotics
and become more difficult to treat).
Q.99) Ans: D
▪ The earth has experienced changes in
climate and land use, such as
Exp: deforestation and intensive farming
• Compulsory Licensing:
practices. Disruptions in environmental
o When the Govt. authorities license companies
conditions and habitats can provide new
or individuals other than the patent owner to opportunities for diseases to pass to
use the rights of the patent — to make, use, animals.
sell or import a product under patent (i.e. a ▪ These changes have led to the spread of
patented product or a product made by a existing or known (endemic) and new or
patented process) — without the permission emerging zoonotic diseases, which are
of the patent owner - it is called Compulsory diseases that can spread between animals
licensing. and people. Examples of zoonotic
▪ Allowed under the WTO’s TRIPS
diseases include:
(intellectual property) Agreement o Salmonella infection
provided certain procedures and o West Nile virus infection
conditions are fulfilled. o Q Fever (Coxiella burnetii)
o The Supreme Court recently asked the Central
o Anthrax
Government why it was not considering o Brucellosis
invoking the powers under the Section 92 of o Lyme disease
the Patents Act, 1970 for compulsory o Ringworm
licensing, for drugs like Remdesivir, o Ebola
Tocilizumab, Favipiravar, which are used for o Rabies
treating COVID-19 patients. o Nipah
▪ Section 92 of the Patents Act is a special
o Severe Acute Respiratory Syndrome
provision enabling the Central o Middle East Respiratory Syndrome
Government to issue compulsory licenses
TEST 1 40
Contact us : info@onlyias.com

OnlyIAS Nothing Else Visit : dpp.onlyias.in


Contact : +91-7007 931 912

▪ The Distemper disease, caused by the


CANINE DISTEMPER VIRUS, is not zoonotic
in nature. Recently, several lions and
leopards succumbed to the CDV in the Gir
Forests of Gujrat.

NEWS: A ‘One Health’ approach that targets people,


animals - The Hindu

TEST 1 41

You might also like